• Shuffle
    Toggle On
    Toggle Off
  • Alphabetize
    Toggle On
    Toggle Off
  • Front First
    Toggle On
    Toggle Off
  • Both Sides
    Toggle On
    Toggle Off
  • Read
    Toggle On
    Toggle Off
Reading...
Front

Card Range To Study

through

image

Play button

image

Play button

image

Progress

1/80

Click to flip

Use LEFT and RIGHT arrow keys to navigate between flashcards;

Use UP and DOWN arrow keys to flip the card;

H to show hint;

A reads text to speech;

80 Cards in this Set

  • Front
  • Back
  • 3rd side (hint)
1. In accordance with the patent laws, rules and procedures as related in the MPEP, for a nonprovisional application to receive a filing date in the USPTO under 37 CFR 1.53(b), all of the following must be filed except:
(A) The basic filing fee required by 37 CFR 1.16(a).
(B) A specification as prescribed by the first paragraph of 35 USC 112.
(C) A description pursuant to 37 CFR 1.71.
(D) At least one claim pursuant to 37 CFR 1.75.
(E) Any drawing required by 37 CFR 1.81(a).
1. ANSWER: (A) is the most correct answer. 35 U.S.C. § 111; 37 CFR § 1.53; MPEP § 601.01. As provided in MPEP § 601.01(a), the filing fee for an application filed under 37 CFR 1.53(b) can be submitted after the filing date. (B), (C), (D) and (E) are incorrect. 37 CFR § 53(b); MPEP § 601.01. 37 CFR § 1.53(b) provides that a filing date is granted on the date on which a specification as prescribed by 35 U.S.C. § 112 containing a description pursuant to 37 CFR § 1.71 and at least one claim pursuant to 37 CFR § 1.75, and any drawing required by 37 CFR § 1.81(a) are filed in the Office. Thus, (B), (C), (D) and (E) are needed to obtain a filing date.
2. A U.S. patent was granted on May 8, 2001. The sole independent claim in the patent is directed to a combination of elements ABCD. A registered practitioner filed a reissue application on April 11, 2003 to narrow sole independent claim. In the reissue application, the independent claim is amended to a combination to elements ABCDE. The reissue application is accompanied by a transmittal letter stating that the application was filed to narrow a claim, that all inventors could not be located to sign the reissue oath or declaration at that time, and that a declaration would be submitted in due course. No other amendments to the claims were filed on April 11, 2003. On May 8, 2003, a declaration signed by all inventors is filed declaring that they had claimed less than they had a right to claim, and that the error arose without deceptive intent. The inventors also filed on May 8, 2003 a preliminary amendment deleting element A from the sole independent claim leaving elements BCDE. The amendment and declaration are filed using the provisions of 37 CFR 1.10. The practitioner included an authorization to charge the practitioner’s deposit account for any necessary fees. Which of the following actions by the primary the examiner in the first Office action is in accordance with the patent laws, rules and procedures as related in the MPEP?
10/15/2003 USPTO Reg. Exam.
3 Afternoon Session (Nbr. 456 Ser. 203)
(A) (B) (C)
(D) (E)
Reject all the claims based upon a broadening reissue outside the two year statutory period authorized by 35 USC 251 since applicant did not file a broadened reissue claim at the time of filing.
Reject all the claims based upon a broadening reissue outside the two year statutory period authorized by 35 USC 251 since applicant did not file a claim to a broadened reissue claim within the two year period set by 35 USC 251.
Reject all the claims based upon a broadening reissue outside the two year statutory period authorized by 35 USC 251 since applicant’s indication in the transmittal letter indicated that the filing of the reissue application was a narrowing reissue and that the broadening amendment was not permissible even if filed within the two-years from the grant of the original patent.
Determine that the application is a proper broadening reissue and perform an examination and issue an Office action in due course.
Determine that the application is a proper broadening reissue and reject the claims under the recapture doctrine since the claims are broader than the issued claims.
2. ANSWER: (D) is the most correct answer. MPEP §§ 1403 and 1412.03, under the heading “When A Broadened Claim Can Be Presented.” A broadening reissue claim must be filed within the two years from the grant of the original patent. (D) is the most correct and the examiner should examine the case as any other application and address appropriate issues concerning reissue examination. See Switzer v. Sockman, 333 F.2d 935, 142 USPQ 226 (CCPA 1964) (a similar rule in interferences). Since applicant filed the amendment by Express Mail, the amendment is treated as being filed with the USPTO on the date of deposit with the US Postal Service. Therefore, (A), (B) and (C) are incorrect answers. A reissue application can be granted a filing date without an oath or declaration, or without the filing fee being present. See 37 CFR § 1.53(f). Applicant will be given a period of time to provide the missing parts and to pay the surcharge under 37 CFR § 1.16(e). See MPEP § 1410.01. Choice (E) is not correct since the mere deletion of an element of a claim does not automatically raise a ground of rejection based on the recapture doctrine. See MPEP § 1412.02.
3. An applicant submits a product-by-process claim to a shoe made by a series of specific process steps. The claim is rejected over a publication under 35 USC 102(b) and 103. Assume for this question that the publication reasonably appears to show the identical shoe, but describes a different method of making the shoe. What is the proper procedure to try to overcome the rejection in accordance with the patent laws, rules and the procedures as related in the MPEP?
(A) Argue that all limitations in the claim must be given weight and that rejection must be withdrawn because the reference does not disclose the claimed method of making steps.
(B) Argue that the examiner has not carried the burden of proving that the shoes are identical.
(C) Present evidence why the steps of the claimed process produce a patentably different structure.
(D) Submit a declaration under 37 CFR 1.132 by the author of the publication describing in more detail how the shoe in the publication was made by a different method.
(E) Argue that the inventor was not aware of the publication when the invention was made.
3. ANSWER: (C) is correct. MPEP §2113, under the heading “Once A Product Appearing To Be Substantially Identical Is Found And A 35 U.S.C. 102/103 Rejection Made, The Burden Shifts To The Applicant To Show An Unobvious Difference,” states “[o]nce the examiner provides a rationale tending to show that the claimed product appears to be the same or similar to that of the prior art, although produced by a different process, the burden shifts to applicant to come forward with evidence establishing an unobvious difference between the claimed product and the prior art product. In re Marosi, 710 F.2d 798, 802, 218 USPQ 289, 292 (Fed. Cir. 1983).”
(A) is incorrect because the patentability of product-by-process claims is based on the product itself. See In re Thorpe, 227 USPQ 964, 966 (Fed. Cir. 1985). Where the end products are the same, the process of making limitations do not have to be given weight in ex parte examination. See Atlantic Thermoplastics Co. v. Faytex Corp., 23 USPQ2d 1481, 1490-91 (Fed. Cir. 1992) (product-by-process claims are treated differently for patentability purposes during ex parte examination in the USPTO than for infringement and validity purposes during litigation). (B) is incorrect because "[o]nce the Examiner provides a rationale tending to show that the claimed product appears to be the same or similar to that of the prior art, although produced by a different process, the burden shifts to applicant to come forward with evidence establishing an unobvious difference between the claimed product and the prior art product." MPEP § 2113. "To the extent that the process limitations distinguish the products over the prior art, they must be given the same consideration as traditional product characteristics." In re Hallman, 210 USPQ 609, 611 (CCPA 1981). Thus, (C) is correct because applicant can show by factual arguments and/or declarations or affidavits under 37 CFR § 1.132 that the

4. A registered practitioner files an international application submission that includes a description, claims and drawings in the United States Receiving Office (RO/US) on Wednesday, January 8, 2003. The submission did not include the required request, international and search fees, or the designation of a PCT contracting State. The RO/US mails an “Invitation to Correct the Purported International Application,” dated January 10, 2003, to the practitioner indicating that the designation of at least one Contracting State, as required by PCT Article 11(1)(iii)(b), was not included. A one-month period for response is set in the Invitation. On Monday, February 10, 2003, the practitioner submits by facsimile a designation sheet of the Request Form designating every available Contracting State, and authorization to charge all required fees. In accordance with the patent laws, rules and procedures as related in the MPEP, will the application be accorded an international filing date?
10/15/2003 USPTO Reg. Exam.
4 Afternoon Session (Nbr. 456 Ser. 203)
(A) (B)
(C) (D) (E)
Yes. The application will be accorded a filing date of January 8, 2003.
Yes. The application will be accorded an international filing date of February 10, 2003.
No. The application will not be accorded an international filing date because the failure to designate at least one contracting State cannot be cured by a facsimile transmission.
No. The application was given a one-month period for response. The practitioner would have had to have filed the response on Friday, February 7, 2003 in order to have been timely.
None of the above.
ANSWER: (C) is the correct answer. 37 CFR §§ 1.6(d)(3) and 1.8(a)(2)(i)(d); MPEP § 502 (reproducing Rule 1.6(d)(3)); MPEP § 512 (reproducing Rule 1.8(a)(2)(i)(d)); and MPEP §1817.01. As stated in MPEP § 1817.01, “[a]ll designations must be made in the international application on filing; none may be added later.” The application will not be accorded an international filing date since the practitioner has tried to cure the failure to designate at least one contracting State by filing a paper using facsimile which is not permitted according to 37 CFR §§ 1.6(d)(3) and 1.8(a)(2)(i)(d). (A) is wrong because applicant has failed to comply with Article 11(1)(iii)(b) on such date. See MPEP § 1810 (reproducing PCT Article 11(1)(iii)(b). (B) is wrong because according to 37 CFR 1.6(d)(3) and 37 CFR 1.8 (a)(2)(i)(d), applicant cannot file an international application by facsimile. See MPEP § 502 (reproducing 37 CFR § 1.6(d)(3)); MPEP § 512 (reproducing 37 CFR § 1.8(a)(2)(i)(d)). Since no designations were included on filing, the application papers cannot be accorded an international filing date. See PCT Article 11(1)(iii)(b). Applicant cannot correct this by filing the designation sheet by facsimile. See MPEP § 502 (reproducing Rule 1.6(d)(3)); MPEP § 512 (reproducing Rule 1.8(a)(2)(i)(d)). (D) is wrong because according to PCT Rule 80.5, when a response is due on a day where the receiving Office is not open for business, applicant has until the next business day. See Appendix T of the MPEP. (E) is incorrect because (C) is correct.
5. In accordance with the patent laws, rules and procedures as related in the MPEP, satisfaction of the written description requirement may not be demonstrated by:
(A) including in the specification a description of an actual reduction to practice.
(B) describing the claimed invention with all of its limitations using such descriptive means as words, structures, figures, diagrams, and formulas that fully set forth the
claimed invention.
(C) describing during prosecution of a new or amended claim an element or limitation
(omitted from the original disclosure in the specification) as an essential or critical
feature of the invention.
(D) including in the specification a description of distinguishing identifying
characteristics sufficient to show that the applicant was in possession of the
claimed invention at the time of filing.
(E) including in the patent application disclosure of drawings or structural chemical
formulas showing that the invention is complete.
ANSWER: (C) is the most correct answer. MPEP §2163, under the heading “GENERAL PRINCIPLES GOVERNING COMPLIANCE WITH THE "WRITTEN DESCRIPTION" REQUIREMENT FOR APPLICATIONS,” and subheading “New or Amended Claims,” states “A claim that omits an element which applicant describes as an essential or critical feature of the invention originally disclosed does not comply with the written description requirement. See Gentry Gallery, 134 F.3d at 1480, 45 USPQ2d at 1503; In re Sus, 306 F.2d 494, 504, 134 USPQ 301, 309 (CCPA 1962) (‘[O]ne skilled in this art would not be taught by the written description of the invention in the specification that any 'aryl or substituted aryl radical' would be suitable for the purposes of the invention but rather that only certain aryl radicals and certain specifically substituted aryl radicals [i.e., aryl azides] would be suitable for such purposes.’).” (A), (B), (D) and (E) are incorrect. Each lists a proper way to demonstrate satisfaction of the written description requirement.
6. Inventor Tip, a scientist in a pencil research laboratory, theorized that, based on the abrasive properties of moon dust, a highly efficient erasure can be made by adding a trace amount of moon dust to a normal pencil erasure formulation. Point, in the Sales department, determined that this would be perfect for a high end product. A U.S. patent application has been filed claiming a pencil erasure formulation with a trace amount of moon dust. An example of how to make the formulation with specified percentages of moon dust is presented therein. Thereafter, Tip learns about the duty to disclose information and he recalls signing a declaration under 37 CFR 1.63 stating that he had reviewed and understood the contents of the specification including the claims. Tip becomes concerned that the use of moon dust was only a theory and that to obtain patent would mislead the public to conclude that moon dust was actually used and found to be effective. The application has been allowed, but the issue fee has not yet been paid. Which of the following is most in accord with patent laws, rules and procedures as related in the MPEP?
10/15/2003 USPTO Reg. Exam.
5 Afternoon Session (Nbr. 456 Ser. 203)
(A) (B)
(C) (D) (E)
Point is under a duty to disclose material information to the USPTO.
Tip is under a duty to disclose his concern regarding the moon rock information to the USPTO.
Both Point and Tip are under a duty to disclose material information to the UPSTO.
There is no duty to disclose information regarding how the moon rock formulation was developed to the USPTO.
Inasmuch as the application is allowed, an appropriate Request for Continued Prosecution pursuant to 37 CFR 1.114 needs to be filed accompanied by a information disclosure regarding the possibility of rejections under 35 USC 101, and 112, first paragraph.
6. ANSWER: (D) is the most correct answer. 37 CFR § 1.56; MPEP § 2001.05. 37 CFR § 1.56(a) sets forth a duty to disclose information that is material to patentability. MPEP § 2001.05 states that “information is not material unless is comes within the definition of 37 CFR 1.56(b)(1) or (b)(2). If information is not material, there is no duty to disclose the information to the Office.” The information that moon dust was never actually used is not material as defined under 37 CFR § 1.56(b)(1) or (2) which state that information is material if “(b)(1) It establishes, by itself or in combination with other information, a prima facie case of unpatentability of a claim; or (2) It refutes, or is inconsistent with, a position the applicant takes in: (i) Opposing an argument of unpatentability relied on b y the Office or, (ii) Asserting an argument of patentability.” That the use of the moon dust as part of an erasure formulation was only theorized and not actually used is acceptable as is an example for making it. MPEP § 608.01(p), II, under the heading “Simulated or Predicted Test Results Or Prophetic Examples,” states that “[s]imulated or predicted test results and prophetical examples (paper examples) are permitted in patent applications. ... Paper examples describe the manner and process of making an embodiment of the invention which has not actually been conducted.” Care, however, must be taken not to state that an experiment was actually run or conducted when it was not and that “[n]o results should be presented as actual results unless they have actually been achieved.” MPEP § 2004, item 8.
7. The claimed invention in a patent application is directed to an explosive composition “comprising 60-90% solid ammonium nitrate, and 10-40% water-in-oil in which sufficient aeration is entrapped to enhance sensitivity to a substantial degree.” The application discloses that the explosive requires both fuel (the ammonium nitrate), and oxygen to “sensitize the composition.” A prior art reference, published more than two years before the effective filing date of the application, discloses explosive compositions containing water-in-oil emulsions having identical ingredients to those claimed, in ranges overlapping with the claimed composition. The only element of the claim not recited in the reference is “sufficient aeration entrapped to enhance sensitivity to a substantial degree.” The reference does not recognize that sufficient aeration sensitizes the fuel to a substantial degree. In addition to the prior art reference, a printed publication contains test data demonstrating that “sufficient aeration” is necessarily an inherent element in the prior art blasting composition under the circumstances. In accordance with the patent laws, rules and the procedures as related in the MPEP, the prior art reference:
(A) anticipates the claim because it discloses every limitation of the claim either explicitly or inherently.
(B) does not anticipate the claim because the prior art reference does not recognize an inherent property.
(C) does not anticipate the claim because the prior art reference does not recognize an inherent function of oxygen.
(D) does not anticipate the claim because the prior art reference does not recognize an inherent ingredient, oxygen.
(E) (B), (C) and (D).
7. ANSWER: (A) is the best answer. 35 U.S.C. § 102; MPEP § 2131.01, under the heading “Extra Reference or Evidence Can Be Used To Show an Inherent Characteristic of the Thing Taught by the Primary Reference,” states “that as long as there is evidence of record establishing inherency, failure of those skilled in the art to contemporaneously recognize an inherent property, function or ingredient of a prior art reference does not preclude a finding of
3
October 15, 2003 Examination Afternoon Session Model Answers
anticipation. Atlas Powder Co. v. IRECO, Inc., 190 F.3d 1342, 1349, 51 USPQ2d 1943, 1948 (Fed. Cir. 1999) (Two prior art references disclosed blasting compositions containing water-in- oil emulsions with identical ingredients to those claimed, in overlapping ranges with the claimed composition. The only element of the claims arguably not present in the prior art compositions was "sufficient aeration . . . entrapped to enhance sensitivity to a substantial degree." The Federal Circuit found that the emulsions described in both references would inevitably and inherently have "sufficient aeration" to sensitize the compound in the claimed ranges based on the evidence of record (including test data and expert testimony).
8. With respect to establishing “reasonable diligence” for under 35 USC 102(g), which of the following statements is or are in accordance with the patent laws, rules and procedures as related in the MPEP?
(1) The inventor and his attorney must drop all other work and concentrate on the particular invention involved.
(2) The entire period during which diligence is required must be accounted for by either affirmative acts or acceptable excuses.
(3) Work relied upon to show reasonable diligence must be directly related to the reduction to practice.
10/15/2003 USPTO Reg. Exam.
6 Afternoon Session (Nbr. 456 Ser. 203)
(A) (B) (C) (D) (E)
Statement (1) only Statement (2) only Statement (3) only Statements (1) and (3) Statements (2) and (3)
8. ANSWER: (E) is the most correct, because statements (2) and (3) are true. The entire period for which diligence is required must be accounted for. MPEP § 2138.06, under the heading “The Entire Period During Which Diligence Is Required Must Be Accounted For By Either Affirmative Acts Or Acceptable Excuses,” states “[a]n applicant must account for the entire period during which diligence is required. Gould v. Schawlow, 363 F.2d 908, 919, 150 USPQ 634, 643 (CCPA 1966) (Merely stating that there were no weeks or months that the invention was not worked on is not enough.).” MPEP § 2138.06, under the heading “Work Relied Upon To Show Reasonable Diligence Must Be Directly Related To The Reduction To Practice,” states “[t]he work relied upon to show reasonable diligence must be directly related to the reduction to practice of the invention in issue. Naber v.Cricchi, 567 F.2d 382, 384, 196 USPQ 294, 296 (CCPA 1977), cert. denied, 439 U.S. 826 (1978). ‘U]nder some circumstances an inventor should also be able to rely on work on closely related inventions as support for diligence toward the reduction to practice on an invention in issue.’” (A) is incorrect because statement (1) is not true – an inventor or his attorney need not drop all other work to establish reasonable diligence. Emery v. Ronden, 188 USPQ 264, 268 (Bd. Pat. Inter. 1974); MPEP § 2138.06. (B) is incorrect because it does not include true statement (3). (C) is incorrect because it does not include true statement (2). (D) is incorrect because it includes false statement (1).
9. Which one of the following statements is in accord with the patent laws, rules and procedures as related in the MPEP regarding double patenting rejections?
(A) A rejection of application claims for obviousness-type double patenting over the claims of a patent which names a different inventive entity (where one inventor is in common with the inventive entity in the application) and was not commonly owned at the time applicant made his or her invention can be overcome with an affidavit or declaration under 37 CFR 1.131 showing that the applicant made the invention in the United States prior to the effective filing date of the patent.
(B) A rejection for obviousness-type double patenting of application claims over patent claims can properly rely on an embodiment which is disclosed in the patent and provides support for the patent claims on which the rejection is based.
(C) The filing of a terminal disclaimer to overcome an obviousness-type double patenting rejection constitutes a binding admission that the rejection is proper.
(D) Application claims are properly rejected for obviousness-type double patenting over claims of a patent having an effective filing date earlier than the effective filing date of the application only if both of the following two conditions are satisfied: (a) the rejected application claims recite an obvious variation of the subject matter recited in the patent claims on which the rejection is based; and (b) the patent claims on which the rejection is based recite an obvious variation of the subject matter recited in the rejected application claims.
(E) None of statements (A) though (D) is correct.
9. CREDIT GIVEN FOR ALL ANSWERS.
10. In accordance with the patent laws, rules and procedures as related in the MPEP, which of the following can a third party submit in a pending published application within two months from the publication date where the submission identifies the application to which it is directed by application number and includes the appropriate fee?
10/15/2003 USPTO Reg. Exam.
7 Afternoon Session (Nbr. 456 Ser. 203)
(A)
(B) (C)
(D) (E)
A list referencing a videotape and copy of the tape showing that the process claimed in the application was in use more than one year before the filing date of the application.
A U.S. patent issued more than one year before the filing date of the application and a written explanation of the patent made by the third party on the patent.
A publication with a publication date more than one year before the filing date of the application and including underlining made by the third party on the publication.
A protest raising fraud and inequitable conduct issues.
A list of the sole Japanese language publication submitted for consideration, including the publication date of the publication, a copy of the Japanese language publication and a written English language translation of the pertinent parts of the publication.
10. ANSWER: (E) is the most correct answer. 37 CFR § 1.99; MPEP § 610. Rule 1.99
provides that a third party may submit in a published application a foreign-language publication and an English language translation of pertinent portions of the publication. The submission must “identify the application to which it is directed by application number,” . . “include the fee set forth in § 1.17(p);” include “a list of the patents or publications submitted for consideration by the Office, including the date of publication of each patent or publication;”. . . a “ copy of each listed patent or publication in written form or at least the pertinent portions;” and an “English language translation of all the necessary and pertinent parts of any non-English language patent or publication in written form relied upon.”
11. A U.S. patent was granted on May 8, 2001 to five inventors. The five inventors assigned their entire patent rights to Q Company. Q Company needs to file a reissue application to broaden the claims of the patent. The registered practitioner preparing the application has been unable to locate any of the five inventors to sign the reissue oath or declaration. Today is May 8, 2003. Which of the following should the practitioner do to enable the applicant to broaden the patent claims in accordance with the patent laws, rules and procedures as related in the MPEP?
(A) Wait to file the reissue application until the first day the signatures of all five inventors can be obtained. At that time, pay the filing fee and file a petition seeking May 8, 2003 as the filing date. File with the petition a showing of the unavailability of all inventors until the filing of the application.
(B) Wait to file the reissue application until the signatures of at least three inventors can be obtained. At that time, file a petition seeking May 7, 2003 as the filing date accompanied by a showing of the unavailability of all inventors on May 8th. Payment of the filing fees may be postponed until receipt of a decision on the petition.
(C) File the reissue application on May 8, 2003, presenting only the claims in the patent, and include a listing of inventors, but not pay the filing fee at the time of filing.
(D) Wait to file the reissue application until the signature of one of the inventors has been obtained since at least one inventor is needed to show a lack of deceptive intent on the part of the applicants.
(E) File the complete reissue application complying with 37 CFR 1.173(a) and 1.53(b) with an unexecuted reissue declaration listing the names of all the inventors with at least one broadening claim on May 8, 2003.
11. ANSWER: (E) is the most correct answer. 35 U.S.C. § 251; 37 CFR § 1.53(f); MPEP § 1403. Filing a broadened reissue application with at least one broadening claim prior to the expiration of the two-year time period set in the statute satisfies the diligence provisions of 35 U.S.C. § 251. The executed reissue oath/declaration and the filing fee may be filed at a later time. According to MPEP § 1403, a reissue application can be granted a filing date without an oath or declaration, or without the filing fee being present. See 37 CFR § 1.53(f). The reissue applicant will be given a period of time to provide the missing parts and to pay the surcharge under 37 CFR § 1.16(e). See MPEP § 1410.01. (A), (B) and (D) are clearly incorrect since the inventors and assignee would be barred from a broadening reissue if filed after the two year period set in the statute. (C) is incorrect since the reissue application was filed without at least one broadening claim prior to the expiration of the two-year time period set in 35 U.S.C. § 251.
12. Which of the following is patentable subject matter under 35 USC 101 in accordance with the patent laws, rules, and procedures as set forth in the MPEP?
10/15/2003 USPTO Reg. Exam.
8 Afternoon Session (Nbr. 456 Ser. 203)
(A) (B)
(C)
(D) (E)
A claim to a new mineral discovered in the earth or a new plant found in the wild. A claim to a method of using a computer to select a set of arbitrary measurement point values. (The selected values are not to be transformed outside of the computer into computer data).
A claim to a method of controlling a mechanical robot which relies upon storing data in a computer that represents various types of mechanical movements of the robot.
A claim to a method of updating alarm limits by changing the number value of a variable to represent the result of the calculation.
A claim to a data structure per se. (The claim does not specify any location where the data structure is stored).
12. ANSWER: (C) is the most correct answer. MPEP § 2106(IV)(B)(2)(b)(i), under the heading “Safe Harbors,” subheading “Independent Physical Acts (Post-Computer Process Activity),” states that “[e]xamples of this type of statutory process include ...[a] method of controlling a mechanical robot which relies upon storing data in a computer that represents various types of mechanical movements of the robot, using a computer processor to calculate positioning of the robot in relation to given tasks to be performed by the robot, and controlling the robot ’s movement and position based on the calculated position.” (A) is a true statement, and is therefore an incorrect answer. As set forth in MPEP § 2105 a “ new mineral discovered in the earth or a new plant found in the wild is not patentable subject matter.” (B) is an incorrect answer.
13. On January 2, 2001, a registered practitioner filed a patent application with the USPTO for inventor Beck. The application includes a specification and a single claim to the invention which reads as follows:
1. Mixture Y made by the process Q1.
In the specification, Mr. Beck discloses that mixture Y has a melting point of 150° F. On June 2, 2001, the practitioner received an Office action from the primary examiner rejecting the claim. The claim is rejected under 35 USC 102/103 as being clearly anticipated by or obvious over Patent A. The examiner states “Patent A teaches mixture Y but made by a different process Q2.” Beck believes he is entitled to a patent to mixture Y. In accordance with the patent laws, rules and procedures as related in the MPEP, which of the following would be the best reply to the rejection of his claim?
(A) An argument that the claimed product has an unexpectedly low melting point of 150° F, supported by an affidavit showing that the mixture Y made by process Q2 exhibits a melting point of 300° F.
(B) An argument that the processes used by applicant and patent A are different, supported by a third-party declaration stating only that the processes are different.
(C) An argument that the claimed product has an unexpectedly low melting point of 150° F, supported by a third-party declaration stating only that the products are different.
(D) An argument that the processes used by applicant and patent A are different, supported by an affidavit showing that the mixture Y made by process Q2 exhibits a melting point of 300° F.
(E) An argument that the claimed product has an unexpectedly low melting point of 150° F because the claimed mixture Y has a melting point of 150° F and the mixture Y of patent A has a melting point of 300° F.
13. ANSWER: (A) is the most correct answer. MPEP § 2113, under the heading “Product- By-Process Claims Are Not Limited To The Manipulations Of The Recited Steps, Only The Structure Implied By The Steps,” states “’even though product-by-process claims are limited by and defined by the process, determination of patentability is based on the product itself. The patentability of a product does not depend on its method of production. If the product in the
5
October 15, 2003 Examination Afternoon Session Model Answers
product-by-process claim is the same as or obvious from a product of the prior art, the claim is unpatentable even though the prior product was made by a different process.’ In re Thorpe, 777 F.2d 695, 698, 227 USPQ 964, 966 (Fed. Cir. 1985).” The issue is whether the claimed mixture Y is the same as or obvious over the patented mixture Y. MPEP § 2113, under the heading “Once A Product Appearing To Be Substantially Identical Is Found And A 35 U.S.C. 102/103 Rejection Made, The Burden Shifts To The Applicant To Show An Unobvious Difference,” states “[o]nce the examiner provides a rationale tending to show that the claimed product appears to be the same or similar to that of the prior art, although produced by a different process, the burden shifts to applicant to come forward with evidence establishing an unobvious difference between the claimed product and the prior art product. In re Marosi, 710 F.2d 798, 802, 218 USPQ 289, 292 (Fed. Cir. 1983).” Evidence that the two processes produce different properties is germane to the issue of patentability of the product-by-process claim. Accordingly, a comparison of the results obtained by conducting the process recited in the claim versus the process used by patent A and which shows that the claimed product exhibits an unexpectedly lower melting point would be a persuasive demonstration that, although the products would appear to be substantially identical, in fact, they are patentably different. Ex parte Gray, 10 USPQ2d 1922 (Bd. Pat. App. & Inter. 1989). Therefore, the best reply to the outstanding rejection would be to argue that the claimed product has an unexpectedly lower melting point and to support that argument with evidence showing that the result of the patent A process is a mixture with higher melting point as compared to the claimed product.
14. Inventor Jones files an application under 35 USC 111(a) on March 27, 2002. The application is a continuation of an international application, which was filed on December 1, 2000. The international application claims priority to a U.S. provisional application filed December 2, 1999. The international application designated the United States, and was published in English under PCT Article 21(2). All applications contained the exact same disclosure. In accordance with the patent laws, rules and procedures as related in the MPEP, what, if any, is the earliest prior art date under 35 USC 102(e) for the publication of the 35 U.S.C. 111(a) application under 35 USC 122(b)?
10/15/2003 USPTO Reg. Exam.
9 Afternoon Session (Nbr. 456 Ser. 203)
(A) (B) (C) (D) (E)
None, the publication has no prior art date under 35 U.S.C. 102(e) March 27, 2002
December 11, 2001
December 1, 2000
December 2, 1999
14. ANSWER: The correct answer is answer (E). See MPEP § 706.02(f)(1) in general and Example 7 in particular. (A) is not correct as the publication under 35 U.S.C. § 122(b) does have a prior art date under 35 U.S.C. 102(e). (B) and (D) are not correct because March 27, 2002 and December 1, 2000 are not the earliest prior art date under 35 U.S.C. § 102(e). The publication has an earlier prior art date than March 27, 2002 and December 1, 2000 because of its benefit/priority claims to the international application and the provisional application. See MPEP § 706.02(f)(1). (C) is not correct as it is not a filing date for any application in this question.
15. Applicant filed an international patent application under the Patent Cooperation Treaty (PCT) designating the United States. A copy of the international application has not been submitted to the USPTO by the International Bureau. The deadline for entering the national stage under 35 USC 371(c) was August 15, 2002. Applicant submitted all of the national stage items required by 35 USC 371(c) by facsimile transmission on August 15, 2002. The facsimile transmission was successfully received by the USPTO on August 15, 2002. The submission included an authorization to charge any required fees to the valid deposit account of the registered practitioner representing applicant. The account contained sufficient funds. Assuming that applicant has made no other national stage submissions under 35 USC 371(c), which of the following statements is most correctly describes why the national stage submission in accordance with the patent laws, rules and the procedures as related in the MPEP is proper or improper?
(A) The national stage submission was proper because facsimile transmission is a valid method of correspondence in the USPTO.
(B) The national stage submission was proper because a copy of an originally executed oath or declaration is acceptable, but the original oath or declaration should be retained as evidence of authenticity.
(C) The national stage submission was improper because a copy of the international application and the basic national fee necessary to enter the national stage as required by 35 USC 371(c) may not be submitted by facsimile transmission.
(D) The national stage submission was improper because the USPTO does not accept fee payments via facsimile transmission.
(E) The national stage submission was improper because facsimile transmission may never be used for PCT applications.
15. ANSWER: (C) is the most correct answer. 37 §§CFR 1.6(d)(3); 1.8(a)(2)(i)(F); 1.495(b); MPEP § 1893.01(a)(1), 2nd paragraph. The filing of the copy of the international application and the basic national fee in order to avoid abandonment under 37 CFR § 1.495(b), as appropriate, may not be transmitted by facsimile. See 37 CFR § 1.6(d)(3) and 37 CFR § 1.8(a)(2)(i)(F). (A) is not the most correct answer because facsimile transmission is not permitted in the situations set forth in 37 CFR § 1.6(d). (B) is not the most correct answer
6
October 15, 2003 Examination Afternoon Session Model Answers
because even though an oath or declaration may be submitted by facsimile transmission as set forth in MPEP § 602, the national stage submission was improper for the reasons discussed in (C). (D) is not the most correct answer because (C) is the most complete answer. Facsimile transmissions may not be used to file a copy of the international application necessary to enter the national stage. (E) is not the most correct answer because facsimile transmission may be used to file certain correspondence in PCT applications. See MPEP § 1805.
16. Which of the following statements is or are in accord with the patent laws, rules and procedures as related in the MPEP?
(1) In a 35 USC 103 obviousness analysis, the proper question is whether the differences between the prior art and the claims would have been obvious to one of ordinary skill in the art.
(2) In a 35 USC 103 obviousness analysis, an inventor’s assertion the he has discovered the source or cause of an identified problem should never be considered.
(3) A 35 USC 103 obviousness analysis requires consideration not just of what is literally recited in the claims, but also of any properties inherent in the claimed subject matter that
are disclosed in the specification.
10/15/2003 USPTO Reg. Exam.
10 Afternoon Session (Nbr. 456 Ser. 203)
(A) (B) (C) (D) (E)
Statement 1 Statement 2 Statement 3 Statements 1 & 2 Statements 1 & 3
16. ANSWER: (C) is the most correct answer. The principle in Statement 3, that consideration of inherent properties is part of proper consideration of the invention as a whole, is recited in MPEP § 2141.02, under the heading “Disclosed Inherent Properties Are Part Of ‘As A Whole’ Inquiry,” and in In re Antonie, 559 F.2d 618, 620, 195 USPQ 6, 8 (CCPA 1977). (A) is incorrect, because the proper question is whether the invention as a whole, not just the differences, would have been obvious. See MPEP § 2141.02, under the heading “The Claimed Invention As A Whole Must Be Considered,” (citing Stratoflex, Inc. v. Aeroquip Corp., 713 F.2d 1530, 218 USPQ 871 (Fed. Cir. 1983). (B) is incorrect because an examiner should consider such assertions by an inventor as part of the “subject matter as a whole.” See MPEP § 2141.02 (citing In re Sponnoble, 405 F.2d 578, 585, 160 USPQ 237, 243 (CCPA 1969)). (D) and (E) are incorrect because they include incorrect Statements 1 and/or 2.
17. A patent application was filed on November 1, 2000 for the invention of J.J. Smithy. The application has no priority or benefit claims to any other application. Claims in the application are separately rejected under 35 USC 102 as being anticipated by each of the following references. Which reference can be properly applied under 35 U.S.C. 102(e) in accordance with the patent laws, rules and procedures as related in the MPEP?
(A) A WIPO publication of an international application under PCT Article 21(2), which has an international filing date of October 3, 2000, was published in English and designated the United States.
(B) A U.S. patent by J.J. Smithy that has a filing date of September 5, 2000.
(C) A U.S. application publication under 35 U.S.C. 122(b) by inventor Jones that was
filed on August 8, 2000.
(D) A journal article by Marks published on October 11, 2000.
(E) All of the above.
17. ANSWER: The correct answer is answer (C). 35 U.S.C. § 102(e); MPEP § 706.02(f). The application publication is a proper reference under 35 U.S.C. 102(e) because it was filed by another prior to the filing date of the invention. See MPEP § 706.02(f) et seq. Answer (A) is incorrect. The reference in answer (A) is not a proper reference under 35 U.S.C. § 102(e) because its international filing date was prior to November 29, 2000 thereby failing one of the three conditions for a WIPO publication of an international application to be applied under 35 U.S.C. § 102(e). See MPEP § 706(f)(1), under the heading “I. Determine The Appropriate 35 U.S.C. 102(e) For Each Potential Reference By Following The Guidelines, Examples And Flow Charts Set Forth Below,” subpart (C), which states “[I]f the potential reference resulted from, or claimed the benefit of, an international application, the following must be determined: (1) If the international application meets the following three conditions: (a) an international filing date on or after November 29, 2000 . . . then the international filing date is a U.S. filing date for prior art purposes under 35 U.S.C. 102(e).” The reference in (B) is not a proper reference under 35 U.S.C. § 102(e) because the reference is not by another. See MPEP § 706.02(f). The reference in (D) is not a proper reference under 35 U.S.C. § 102(e) because 35 U.S.C. § 102(e) refers to patents and patent applications, not journal articles. See MPEP § 706.02(f)(1). (E) is not correct because (C) is correct and (A), (B) and (D) are incorrect.
18. A registered practitioner filed a design patent application on December 30, 2003. The application was filed with an inventor-executed declaration naming Jon Jones as the sole inventor, who has not assigned the invention and is not under an obligation to assign his invention. The filing receipt was recently received, indicating that the application will be published on Thursday, July 1, 2004. In reviewing the filing receipt the practitioner realizes that the typed name of the inventor contained a typographical error (an “h” was missing) and that the correct spelling was John Jones. Which of the following would be the course of action at the least expense to correct the error in accordance with the patent laws, rules and procedures as related in the MPEP?
(A)
(B) (C)
(D) (E)
The practitioner should file a request under 37 CFR 1.48 to correct the inventorship of the application with a new declaration under 37 CFR 1.63 signed by John Jones (with the correct spelling of this name), a statement by Mr. Jones as to how the error occurred and that the error was without deceptive intention, and the processing fee set forth in 37 CFR 1.17(q).
The practitioner should file a petition under 37 CFR 1.182 and the petition fee set forth in 37 CFR 1.17(h), requesting correction of the spelling of the inventor’s name.
The practitioner should file a request for a corrected filing receipt and a separate letter to the Office explaining that the declaration contains a typographical error, that the correct spelling of the inventor’s name is John Jones, and requesting correction of the Office records.
The practitioner should expressly abandon the application, and file a continuation with a new declaration with the correct spelling.
The practitioner should call the examiner and tell the examiner that the inventor’s name is wrong, and ask for the examiner to change the name on the declaration.
18. ANSWER: (C) is the most correct answer. See MPEP § 605.04(b), which states “Except for correction of a typographical or transliteration error in the spelling of an inventor's name, a request to have the name changed to the signed version or any other corrections in the name of the inventor(s) will not be entertained...When a typographical or transliteration error in the spelling of an inventor's name is discovered during pendency of an application, a petition is not required, nor is a new oath or declaration under 37 CFR 1.63 needed. The U.S.[PTO] should simply be notified of the error and reference to the notification paper will be made on the previously filed oath or declaration by the Office.” (A), (B) and (D) could result in the spelling of Jon’s name being corrected in USPTO records, but would do so at a higher cost to applicant,
19. The claims in an application are rejected under 35 USC 103 as obvious over prior art reference A in view of prior art reference B. All of the following statements are in accord with the patent laws, rules and procedures as related in the MPEP except:
(A) Where the combination of prior art references provides motivation to make the claimed invention to achieve the same advantage or result discovered by the applicant, the references do not have to expressly suggest the combination of references.
(B) The rationale to modify or combine the prior art references may be reasoned from knowledge generally available to one of ordinary skill in the art, established scientific principles, or legal precedent established by prior case law.
(C) In considering the disclosure of the prior art references, it is proper to take into account the specific teachings of the references, as well as the inferences that one skilled in the art could reasonably draw from the specific teachings.
(D) An examiner may take official notice of facts outside the record that are capable of instant and unquestionable demonstration as being “well known” prior art or common knowledge in the art.
(E) To rely on equivalence as a rationale supporting an obviousness rejection under 35 USC 103 an examiner may base the rejection on the mere fact that the components at issue are functional or mechanical equivalents.
19. ANSWER: (E) is the most correct answer. MPEP § 2144.06, under the heading “Substituting Equivalents Known For The Same Purpose,” states “[i]n order to rely on equivalence as a rationale supporting an obviousness rejection, the equivalency must be recognized in the prior art, and cannot be based on applicant's disclosure or the mere fact that the components at issue are functional or mechanical equivalents. In re Ruff, 256 F.2d 590, 118 USPQ 340 (CCPA 1958).”
20. Recommend which of the following rejections under 35 USC 102 in a reexamination proceeding is in accordance with the patent laws, rules and procedures as related in the MPEP.
10/15/2003 USPTO Reg. Exam.
12 Afternoon Session (Nbr. 456 Ser. 203)
(A) (B)
(C)
(D) (E)
A rejection under 35 USC 102(a) based on an affidavit that the invention was known or used by others before the invention thereof by the applicant for patent. A rejection under 35 USC 102(b) based on an affidavit that the invention was in the public use in this country more than one year prior to the date of the application for a patent in the United States.
A rejection under 35 USC 102(e) that the invention was described in a patent by another filed in the United States before the invention thereof by the patent applicant.
A rejection under 35 USC 102(f) based on an affidavit that the applicant did not himself invent the subject matter sought to be patented.
A rejection under 35 USC 102(b) that the invention was on sale in this country, more than one year prior to the date of the application for patent in the United States.
20. ANSWER: (C) is the most correct answer. 35 U.S.C. § 302; 37 CFR § 1.552; and MPEP § 2258. MPEP § 2258, under the heading “Prior Patents Or Printed Publications,” states “[r]ejections on prior art in reexamination proceedings may only be made on the basis of prior patents or printed publications. Prior art rejections may be based upon the following portions of 35 U.S.C. 102: . . .(e).” (A), (B), (D) and (E) are incorrect. MPEP § 2258, under the heading “Matters Other Than Patents or Printed Publications,” states “[r]ejections will not be based on matters other than patents or printed publications, such as public use or sale, inventorship, 35 U.S.C. 101, fraud, etc. In this regard, see In re Lanham, 1 USPQ2d 1877 (Comm'r Pat. 1986), and Stewart Systems v. Comm'r of Patents and Trademarks, 1 USPQ2d 1879 (E.D. Va. 1986). A rejection on prior public use or sale, insufficiency of disclosure, etc., cannot be made even if it relies on a prior patent or printed publication. Prior patents or printed publications must be applied under an appropriate portion of 35 U.S.C. 102 and/or 103 when making a rejection.” Reexamination is limited to substantially new questions of patentability based on patents and publications.
21. Which of the following would comply with the patent laws, rules and procedures as related in the MPEP and would be a fully responsive reply to a non-final Office action on the merits rejecting all the claims in the application as being unpatentable under 35 USC 102 and/or
103 over prior (A)
(B)
(C)
(D) (E)
art references?
A timely filed and properly signed written reply which does not include an amendment to the claims, but includes a request for the examiner’s rejections to be reconsidered supported by arguments replying to every ground of rejection and distinctly and specifically points out the supposed errors in every rejection. and pointing out the specific distinctions believed to render the claims patentable over any applied references.
A timely filed and properly signed written reply which includes an amendment canceling all the claims in the application and adding new claims, and a request for the examiner’s rejections to be reconsidered in view of the newly presented claims.
A timely filed and properly signed written reply which does not include an amendment to the claims, but does generally alleges that the claims define a patentable invention.
A timely filed and properly signed written request for continued examination (RCE).
All of the above.
21. ANSWER: (A) is the most correct answer. 37 CFR § 1.111; MPEP § 714.02. Section 1.111 states in pertinent part: “(a)(1) If the Office action after the first examination (§ 1.104) is adverse in any respect, the applicant or patent owner, ... must reply and request reconsideration or further examination, with or without amendment. ... (b) In order to be entitled to reconsideration or further examination, the applicant or patent owner must reply to the Office action. The reply by the applicant or patent owner must be reduced to a writing which distinctly and specifically points out the supposed errors in the examiner’s action and must reply to every ground of objection and rejection in the prior Office action. The reply must present arguments pointing out the specific distinctions believed to render the claims, including any newly presented claims, patentable over any applied references. ...The applicant’s or patent owner’s reply must appear throughout to be a bona fide attempt to advance the application or the reexamination proceeding to final action. A general allegation that the claims define a patentable invention without specifically pointing out how the language of the claims patentably distinguishes them from the references does not comply with the requirements of this section.” MPEP § 714.02 states “In all cases where reply to a requirement is indicated as necessary to further consideration of the claims ... a complete reply must either comply with the formal requirements or specifically traverse each one not complied with.”
22. Which, if any, of the following statements is in accord with the patent laws, rules and procedures as related in the MPEP?
10/15/2003 USPTO Reg. Exam.
13 Afternoon Session (Nbr. 456 Ser. 203)
(A)
(B)
(C) (D)
(E)
Where an inventor's residence is stated correctly in the 37 CFR 1.76 application data sheet and incorrectly in the inventor's 37 CFR 1.63 oath or declaration, the discrepancy must be corrected by filing a supplemental 37 CFR 1.67 oath or declaration giving the correct residence.
Where two inventors file separate 37 CFR 1.63 oaths or declarations which do not identify both inventors, the USPTO will presume they are joint inventors and will not require new oaths or declarations.
A dependent claim which merely repeats a limitation that appears in the claim on which it depends is properly rejected under the fourth paragraph of 35 USC 112. In a statement under 37 CFR 1.97(e)(1) specifying that “each item of information contained in the information disclosure statement was first cited in any communication from a foreign patent office in a counterpart foreign application not more than three months prior to the filing of the statement,” the three-month period begins on the date the communication was first received by either a foreign associate or a U.S. registered practitioner.
None of statements (A) to (D) is correct.
22. ANSWER: (E) is the correct answer. (A) is not in accord the patent laws, rules and procedures as related in the MPEP. Regarding (A), MPEP § 601.05, states (“[i]f an application is filed with an application data sheet correctly setting forth the citizenship of inventor B, and an executed 37 CFR 1.63 declaration setting forth a different incorrect citizenship of inventor B, the Office will capture the citizenship of inventor B found in the application data sheet. Applicant, however, must submit a supplemental oath or declaration under 37 CFR 1.67 by inventor B setting forth the correct citizenship even though it appears correctly in the application data sheet.
23. A patent application is filed having one claim (claim 1). The most relevant prior art uncovered by the primary examiner, after searching the claimed subject matter, is a published abstract summarizing the disclosure of a foreign patent document. The abstract is in English, the foreign document is in German. Both the published abstract and the foreign document are prior art under 35 USC 102(b). The published abstract provides an adequate basis for concluding that claim 1 is prima facie obvious under 35 USC 103. Which of the following actions is in accord with the patent laws, rules and procedures as related in the MPEP?
(A) Reject claim 1 under 35 USC 103, based on the abstract, because it is a publication in its own right and provides sufficient basis for a prima facie case of obviousness.
(B) Reject claim 1 under 35 USC 103, based on the abstract, because disclosures that are not in English cannot form the basis of a prior art rejection.
(C) Reject claim 1 under 35 USC 103, based on the German-language patent document, as evidenced by the abstract.
(D) Do not reject claim 1 based on the abstract; instead, obtain a translation of the German-language document and determine whether its full disclosure supports a rejection under 35 USC 102(b) or 103.
(E) Do not reject the claims based on the abstract because an abstract can never provide sufficient disclosure to be enabling.
23. ANSWER: (A) or (D) is accepted as correct. As to (A), MPEP § 706.02, under the heading “Reliance Upon Abstracts And Foreign Language Documents In Support Of A Rejection,” states “[i]n limited circumstances, it may be appropriate for the examiner to make a rejection in a non-final Office action based in whole or in part on the abstract only without relying on the full text document.” In the facts, the “published abstract provides an adequate basis for concluding that claim 1 is prima facie obvious under 35 USC 103.” As to (D) MPEP § 706.02, under the heading “Reliance Upon Abstracts And Foreign Language Documents In Support Of A Rejection,” states “[c]itation of and reliance upon an abstract without citation of and reliance upon the underlying scientific document is generally inappropriate where both the abstract and the underlying document are prior art. See Ex parte Jones, 62 USPQ2d 1206, 1208 (Bd. Pat. App. & Int. 2001) (unpublished).”
24. Applicant filed a provisional patent application in the USPTO under 35 USC 111(b) on Tuesday, November 30, 1999. On Tuesday, November 28, 2000, applicant filed a nonprovisional application in the USPTO under 35 USC 111(a) that properly claimed priority under 35 USC 119(e) to the filing date of the provisional application. On Wednesday, November 29, 2000, applicant filed an international application for patent in the USPTO under the Patent Cooperation Treaty that designated the United States and properly claimed priority to both the provisional and the nonprovisional applications. On Friday, July 28, 2001, applicant filed a national stage application in the USPTO under 35 USC 371, providing all of the requirements under 35 USC 371 and properly claiming benefit to the filing date of the provisional application under 35 USC 119(e) and the nonprovisional application under 35 USC 120. The national stage application was published on Tuesday, January 30, 2002 and issued as a patent on Tuesday, February 4, 2003. Assuming no patent term extension or adjustment, the patent term ends on the date that is 20 years from which of the following dates in accordance with the patent laws, rules and procedures as related in the MPEP?
10/15/2003 USPTO Reg. Exam.
14 Afternoon Session (Nbr. 456 Ser. 203)
(A) (B) (C) (D) (E)
Tuesday, November 30, 1999 Tuesday, November 28, 2000 Wednesday, November 29, 2000 Friday, July 28, 2001
Tuesday, February 4, 2003
24. ANSWER: The filing date of the nonprovisional application, (B), is the correct answer. See MPEP § 201.04(b), which states “[t]he [Uruguay Agreement Round Act] provides a mechanism to enable domestic applicants to quickly and inexpensively file provisional applications. Under the provisions of 35 U.S.C. § 119(e) applicants are entitled to claim the benefit of priority in a given application in the United States. The domestic priority period will not count in the measurement of the 20-year patent term. See 35 U.S.C. 154(a)(3). Thus, domestic applicants are placed on the same footing with foreign applicants with respect to the patent term.” A provisional application is filed under 35 U.S.C. § 119(e) and according to 35 U.S.C. § 154(a)(3), such a filing date is not taken into account in determining patent term. Therefore, (A) is incorrect.
25. A registered practitioner files a nonprovisional utility application in 2000. In 2002, the practitioner files a continuation-in-part application and claims benefit of the filing date of the 2000 application for the 2002 application. Thereafter, the practitioner amends the 2002 application to include claims that were not present in either the originally filed 2000 application or the originally filed 2002 application. The primary examiner properly concludes that the added claims are not supported by the original disclosure in either application. Which of the following is in accord with the patent laws, rules and procedures as related in the MPEP?
(A) The added claims are rejected for lack of written description under 35 USC 112, first paragraph.
(B) The added claims are rejected as new matter under 35 USC 132.
(C) The added claims are denied benefit of the filing date of the 2000 application.
(D) (A) and (B).
(E) (A) and (C).
25. ANSWER: (E) is the most correct answer. Both (A) and (C) are correct. MPEP § 2163.01, under the heading “Support For The Claimed Subject Matter In The Disclosure,” states that “[I]f the examiner concludes that the claimed subject matter is not supported [described] in an application as filed, this would result in a rejection of the claim on the ground of a lack of written description under 35 U.S.C. 112, first paragraph, or denial of the benefit of filing date of a previously filed application.” (B) is incorrect. MPEP § 2163.01 states that unsupported claims “should not be rejected or objected to on the ground of new matter. As framed by the court in In re Rasmussen, 650 F.2d 1212, 211 USPQ 323 (CCPA 1981), the concept of new matter is properly employed as a basis for objection to amendments to the abstract, specification or drawings attempting to add new disclosure to that originally presented.” (D) is incorrect because (B) is incorrect.
26. Which of the following best describes a situation for which a reply to the examiner’s Office action including both an affidavit filed under 37 CFR 1.131 and an affidavit filed under 37 CFR 1.132 may be in accord with the patent laws, rules and procedures as related in the MPEP?
10/15/2003 USPTO Reg. Exam.
15 Afternoon Session (Nbr. 456 Ser. 203)
(A) (B)
(C)
(D)
(E)
In a timely reply to a non-final Office action, where the examiner’s sole rejection of appellant’s claims is based on an alleged violation of the enablement requirement of 35 USC 112.
In a timely reply to non-final Office action, where the examiner’s sole rejection of appellant’s claims is a rejection under 35 USC 103(a) employing a non-patent document that was published less than one year prior to the filing date of appellant’s patent application.
In a timely reply to a non-final Office action, where the examiner’s sole rejection of appellant’s claims is a rejection under 35 USC 103(a) employing a non- commonly owned U.S. patent as prior art under 35 USC 102 (e) that claims the same invention as applicant.
In a timely reply to an examiner’s answer presenting the affidavits for the first time, where in the examiner’s first Office action and final rejection, the examiner maintains the same rejection under 35 USC 103(a) of all of appellant’s claims based in part on a non-patent document that was published less than one year prior to the filing date of appellant’s patent application.
In a timely reply to a final Office action presenting the affidavits for the first time, where in the examiner’s first Office action, the examiner’s sole rejection of appellant’s claims is a rejection under 35 USC 103(a) employing a non-patent document that was published less than one year prior to the filing date of appellant’s patent application.
26. ANSWER: (B) is the most correct answer. MPEP § 715, under the heading “Situations Where 37 CFR 1.131 Affidavits or Declarations Can Be Used,” provides that an affidavit under
11
October 15, 2003 Examination Afternoon Session Model Answers
37 CFR § 1.131 may be used to antedate a reference that qualifies as prior art under 35 U.S.C. § 102(a) but not 35 U.S.C. § 102(b). Also, MPEP § 716 provides that objective evidence traversing a rejection may be presented in a timely submitted affidavit under 37 CFR § 1.132. As for (A), an affidavit under 37 CFR § 1.131 would not serve any useful purpose in those situations. See MPEP § 715. Regarding (C), an affidavit under 37 CFR § 1.131 would be inappropriate. See MPEP §715, under the heading “Situations Where 37 CXFR 1.131 Affidavits or Declarations Are Inappropriate.” As for (D) and (E), a reply including affidavits under 37 CFR § 1.131 and 37 CFR § 1.132 normally would be considered untimely under the circumstances set forth in those answers. See 37 CFR §§ 1.116, 1.192(a) and 1.195. Also, see MPEP §§ 715.09 and 716.01. Consequently, (B) is the most correct answer.
27. In accordance with the patent laws, rules and procedures as related in the MPEP, which of the following is not within the scope of the term “on sale” as it is used in 35 USC 102(b)?
(A) A sale conditioned on buyer satisfaction.
(B) A sale that did not result in a profit.
(C) A single sale of the claimed subject matter.
(D) A commercial offer to sale the claimed subject matter.
(E) An offer to sale the patent rights in the claimed subject matter.
27. ANSWER: (E) is the most correct answer. As set forth in MPEP § 2133.03(b), under the heading “I The Meaning Of ‘Sale’,” subheading “A Sale of Rights Is Not a Sale of the Invention and Will Not in Itself Bar a Patent,” “[a]n assignment or sale of the rights, such as patent rights, in the invention is not a sale of ‘the invention’ within the meaning of section 102(b). The sale must involve the delivery of the physical invention itself. Moleculon Research Corp. v. CBS, Inc., 793 F.2d 1261, 1265, 229 USPQ 805, 809 (Fed. Cir. 1986).” (A) is incorrect because it is within the scope of “on sale” as it is used in 35 U.S.C. § 102(b). MPEP § 2133.03(b), under the heading “I. The Meaning Of ‘Sale’,” subheading “A. Conditional Sale May Bar a Patent,” states “[a]n invention may be deemed to be ‘on sale’ even though the sale was conditional. The fact that the sale is conditioned on buyer satisfaction does not, without more, prove that the sale was for an experimental purpose. Strong v. General Elec. Co., 434 F.2d 1042, 1046, 168 USPQ 8, 12 (5th Cir. 1970).”
28. A patent application is filed disclosing and claiming a system for detecting expired parking meters. The specification fully supports the original, sole claim. The application discloses that the “electronics control unit” contains a comparator and an alarm. The application includes several drawings. One of the drawings shows a block diagram of the system, illustrating the electronics control unit as a box, labeled “electronics control unit.” The sole claim of the application is as follows:
The claim. A system for detecting expired parking meters, comprising: a timer mechanism; an infrared sensor for detecting the presence of a parked vehicle; and an electronics control unit, including a comparator and an alarm, coupled to the infrared sensor and the timer mechanism.
A final Office action, dated February 3, 2004, indicates that the sole claim contains allowable subject matter, but includes an objection to the specification, on the grounds that the subject matter of the electronics control unit, though described in a sufficiently specific and detailed manner in the original specification, was required to be shown in the drawings under 37 CFR 1.83. The Office action did not set a period for reply. Determine which of the following actions, if any, comports with the patent laws, rules and procedures as related in the MPEP for overcoming the objection.
10/15/2003 USPTO Reg. Exam.
16 Afternoon Session (Nbr. 456 Ser. 203)
(A) (B) (C)
(D) (E)
On April 1, 2004, a Notice of Appeal is filed together with appropriate fees, and a brief pointing out that a patent should issue since the subject matter of the electronics control unit was adequately described in the original specification.
On April 1, 2004, a drawing is filed in the USPTO illustrating only the comparator and alarm of the electronics control unit that was described in the original specification.
On April 1, 2004, a Notice of Appeal of appeal is filed together with appropriate fees, and a brief pointing out that the addition of a drawing showing the electronics control unit would not constitute addition of new matter since the electronics control unit was adequately described in the original specification.
On September 1, 2004, a petition is filed urging that no further drawing should be required because the subject matter of the electronics control unit, for purposes of the application, was adequately disclosed in the block diagram drawing.
None of the above.
28. ANSWER: (B). 37 CFR §1.83(a); MPEP §§608.01(l) and 706.03(o). MPEP §608.01(l) states “[w]here subject matter not shown in a drawing...is claimed in the specification as filed, and such original claim itself constitutes a clear disclosure of this subject matter, then the claim should be treated on its merits, and requirement made to amend the drawing and description to show this subject matter...It is the drawing...that [is] defective, not
12
October 15, 2003 Examination Afternoon Session Model Answers
the claim. It is, of course, to be understood that this disclosure in the claim must be sufficiently specific and detailed to support the necessary amendment of the drawing...” MPEP § 608.02(d) and 706.03(o). MPEP § 706.03(o), state “[i]f subject matter capable of illustration is originally claimed and it is not shown in the drawing, the claim is not rejected but applicant is required to add it to the drawing. See MPEP § 608.01(l).” (A), (C), and (D) are incorrect.
29. On Thursday, February 6, 2003, applicant files an application for a design patent in Country X, which issues the patent on the filing date. In accordance with the patent laws, rules and the procedures as related in the MPEP, what is the last date applicant can file a U.S. design application to avoid any loss of patent rights?
(A) Friday, February 6, 2004 (assume not a Federal holiday).
(B) Thursday, February 5, 2004 (assume not a Federal holiday).
(C) Wednesday, August 6, 2003.
(D) Wednesday, May 6, 2003.
(E) None of the above are correct.
29. ANSWER: The correct answer is (C). See 35 U.S.C. § 172; MPEP § 1504.02. 35 U.S.C. § 172 provides that the time specified in 35 U.S.C. 102(d) shall be six months in the case of designs. Thus, to avoid a statutory bar under 35 U.S.C. § 102(d), the U.S. design patent application must be made within six months of the foreign filing, i.e., by August 6, 2003. MPEP § 1504.02 states “[r]egistration of a design abroad is considered to be equivalent to patenting under 35 U.S.C. 119(a)-(d) and 35 U.S.C. 102(d), whether or not the foreign grant is published. (See Ex parte Lancaster, 151 USPQ 713 (Bd. App. 1965);
30. Co-inventors Smith and Jones filed an application for a patent on a cell phone, on May 15, 2002. They received a first Office action from a primary examiner rejecting the claims under 35 USC 102(a) over a publication by Bell and Watson, published on April 5, 2002, describing a cell phone having all the same features as is claimed in the patent application. In reply, the co-inventors each submitted a declaration under 37 CFR 1.131 stating that they had actually reduced the invention to practice no later than March 13, 2002. However, the declarations failed to include two claimed features. Neither the particular antenna needed to enable the cell phone could receive transmissions from the local cellular transmitting tower, nor a detachable carrying strap was included in the declarations. As evidence of their prior reduction to practice, Smith and Jones submitted their co-authored journal article. The journal article contained a figure of the cell phone as described in the declarations. That is, the cell phone shown in the figure of the article lacked an antenna and a detachable strap. The article was received by the journal on March 13, 2002, and was published on April 30, 2002. The cell phones shown in the figure in the Bell and Watson publication, and in the Smith and Jones patent application have the particular antenna and a detachable strap. Which of the following actions, if taken by the examiner, would be the most proper in accordance with the patent laws, rules and the procedures as related in the MPEP?
10/15/2003 USPTO Reg. Exam.
17 Afternoon Session (Nbr. 456 Ser. 203)
(A) (B) (C) (D)
(E)
The examiner should maintain the rejection of the claims under 35 USC 102(a) and make the rejection final.
The examiner should withdraw the rejection and look for references which have a publication date prior to May 15, 2001.
The examiner should withdraw the rejection and notify Smith and Jones that their application is in condition for allowance.
The examiner should maintain the rejection, but indicate that the claims would be allowable if Smith and Jones provided an original copy of the figure published in their journal article as factual support for their declarations.
The examiner should maintain the rejection and inform Smith and Jones that the declarations are insufficient because they cannot “swear behind” a reference which is a statutory bar.
30. ANSWER: (A) is the correct answer. MPEP (8th Ed.) § 715.07, under the heading “Facts and Documentary Evidence” states that “The essential thing to be shown under 37 CFR 1.131 is priority of invention and this may be done by any satisfactory evidence of the fact. FACTS, not conclusions, must be alleged. Evidence in the form of exhibits may accompany the affidavit or declaration. ... The affidavit or declaration must state FACTS and produce such documentary evidence and exhibits in support thereof as are available to show conception and completion of invention in this country or in a NAFTA or WTO member country (MPEP § 715.07(c)) at least the conception being at a date prior to the effective date of the reference. ... In general, proof of actual reduction to practice requires a showing that the apparatus actually existed and worked for its intended purpose.” Here, the co-inventors admit, and the documentary exhibits relied upon demonstrate that they failed to reduce the claimed invention to practice prior to the publication date of the Bell and Watson reference. It is also apparent that due to the lack of an antenna in the cell phone described in Smith’s and Jones’s declarations and journal article, that the cell phone which was reduced to practice prior to the publication date of
13
October 15, 2003 Examination Afternoon Session Model Answers
the Bell and Watson article would not have worked for its intended purpose. Accordingly, the examiner should maintain the rejection and make it final. (B) and (C) are incorrect choices since the evidence of record shows that Smith and Jones are unable to overcome the prior art. (D) is wrong because an original copy of the published figure which shows that Smith and Jones were not in possession of the claimed invention prior to Bell and Watson publication cannot help their case. (E) is incorrect because prior art under 102(a) is not a statutory bar.
31. In accordance with the patent laws, rules and procedures as related in the MPEP, for a nonprovisional application to receive a filing date in the USPTO under 37 CFR 1.53(b), all of the following must be filed except:
(A) An oath or declaration executed by applicant pursuant to 37 CFR 1.63.
(B) A specification as prescribed by the first paragraph of 35 USC 112.
(C) A description pursuant to 37 CFR 1.71.
(D) At least one claim pursuant to 37 CFR 1.75.
(E) A drawing when required by 37 CFR 1.81(a).
31. ANSWER: (A) is the most correct answer. 35 U.S.C. § 111; 37 CFR § 1.53; MPEP § 601.01 As provided in 37 CFR § 1.53(f) and MPEP § 601.01(a), the oath or declaration for an application filed under 37 CFR 1.53(b) can be submitted after the filing date. (B), (C), (D) and (E) are incorrect. 37 CFR § 53(b); MPEP § 601.01. 37 CFR § 1.53(b) provides that a filing date is granted on the date on which a specification as prescribed by 35 U.S.C. § 112 containing a description pursuant to § 1.71 and at least one claim pursuant to § 1.75, and any drawing required by § 1.81(a) are filed in the Office. Thus, (B), (C), (D) and (E) are needed to obtain a filing date.
32. Determine which of the following documents, if any, must also contain a separate verification statement in accordance with the patent laws, rules and procedures as related in the MPEP.
(A) A request to correct inventorship in a pending application.
(B) A petition to make an application special.
(C) A claim for foreign priority.
(D) A substitute specification.
(E) None of the above.
32. ANSWER: (E) is the most correct answer. MPEP § 410 states that the certification requirement set forth in 37 CFR § 10.18(b) “has permitted the USPTO to eliminate the separate verification requirement previously contained in 37 CFR ...1.48 [correction of inventorship in a patent application], ...1.55 [claim for foreign priority], ...1.102 [petition to make an application special], [and] ... 1.125 [substitute specification].”
33. A registered practitioner files an application on the client’s discovery that adding silica to a known plastic composition containing the flame retardant, X, results in increased flame retardance. The application claims a composition comprising the known plastic composition containing X and also silica. The primary examiner rejects the claim on the basis that applicant admits that X was a known flame retardant and that there is no evidence of improved flame retardance. In accordance with the patent laws, rules and procedures as related in the MPEP, a proper reply could include which of the following argument(s) to rebut and overcome the rejection?
10/15/2003 USPTO Reg. Exam.
18 Afternoon Session (Nbr. 456 Ser. 203)
(A) (B)
(C)
(D) (E)
The examiner cannot rely on admitted prior art.
The examiner has not shown that the prior art appreciated applicant’s discovery of silica to be a flame retardant.
The examiner has not made out a prima facie case of obviousness due to lack of motivation in the prior art or in the knowledge generally available to one of ordinary skill in the art for adding silica to the known plastic composition.
The applicant does not have to show an improved or unexpected result for the claimed invention.
(C) and (D).
33. ANSWER: (E) is the most correct answer. MPEP §§ 2141, 2142, 2143 and 2143.01. Regarding (C) MPEP 2142 under the heading “ESTABLISHING A PRIMA FACIE CASE OF OBVIOUSNESS,” states “To establish a prima facie case of obviousness, three basic criteria must be met. First, there must be some suggestion or motivation, either in the references themselves or in the knowledge generally available to one of ordinary skill in the art, to modify the reference or to combine reference teachings. Second, there must be a reasonable expectation of success. Finally, the prior art reference (or references when combined) must teach or suggest all the claim limitations. The teaching or suggestion to make the claimed combination and the reasonable expectation of success must both be found in the prior art, and not based on applicant’s disclosure. In re Vaeck, 947 F.2d 488, 20 USPQ2d 1438 (Fed. Cir. 1991). See MPEP § 2143 for decisions pertinent to each of these criteria.” MPEP § 2143 states the same criteria, and further “The teaching or suggestion to make the claimed combination and the reasonable expectation of success must both be found in the prior art, not in applicant's disclosure. In re Vaeck, 947 F.2d 488, 20 USPQ2d 1438 (Fed. Cir. 1991).” Regarding motivation, MPEP 2143.01 states “Obviousness can only be established by combining or modifying the teachings of the prior art to produce the claimed invention where there is some teaching, suggestion, or motivation to do so found either explicitly or implicitly in the references themselves or in the knowledge generally available to one of ordinary skill in the art. ‘The test for an implicit showing is what the combined teachings, knowledge of one of ordinary skill in the art, and the nature of the problem to be solved as a whole would have suggested to those of ordinary skill in the art.’
34. In accordance with the patent laws, rules and procedures as related in the MPEP , which of the following paper is precluded from receiving the benefit of a certificate of mailing or transmission under 37 CFR 1.8?
(A) An amendment, replying to an Office action setting a period for reply, transmitted by mail with a certificate of mailing to the USPTO from a foreign country.
(B) An amendment, replying to an Office action setting a period for reply, transmitted
by facsimile with a certificate of transmission to the USPTO from a foreign
country.
(C) An information disclosure statement (IDS) under 37 CFR 1.97 and 1.98
transmitted after the first Office action.
(D) A request for continued examination (RCE) under 37 CFR 1.114.
(E) An appeal brief.
34. ANSWER: (A) is the most correct answer. See MPEP § 512, which states “The Certificate of Mailing procedure does not apply to papers mailed in a foreign country.” (B) is not correct. See MPEP § 512. Certificate of transmission procedure applies to correspondence transmitted to the Office from a foreign country and an amendment is not prohibited from being transmitted by facsimile and is not precluded from receiving the benefits under 37 CFR § 1.8. (C) is not correct. See MPEP § 609, subsection “Time for Filing.” An IDS will be considered to have been filed on the date of mailing if accompanied by a properly executed certificate of mailing or facsimile transmission under 37 CFR § 1.8. (D) is not correct. See MPEP § 706.07(h) Comparison Chart. An RCE is entitled to the benefit of a certificate of mailing or transmission under 37 CFR § 1.8. (E) is not correct. See MPEP § 1206. An appeal brief is entitled to the benefit of a certificate of mailing or transmission under 37 CFR § 1.8 because it is required to be filed in the Office within a set time period which is 2 months from the date of appeal.
35. The sole claim in an application filed by A and having an effective filing date of June 5, 2002, recites an electrical signal amplifier comprising a plurality of germanium transistors connected together in a particular configuration. The claim is rejected under 35 USC 103(a) as being obviousness over a primary nonpatent reference publication (Reference P) in view of a secondary nonpatent reference publication (Reference S). Reference P has an effective date of April 3, 2002, and names A and B as the authors. Reference S has an effective date of December 10, 2001, and names C as the sole author. Reference P discloses an electrical signal amplifier including a plurality of silicon transistors connected together in the same configuration as that set forth in the claim. Reference S discloses a signal amplifier employing germanium transistors connected in a configuration different from the claimed configuration. The applicant does not deny that the references render the claimed subject matter prima facie obvious. Which, if any, of the declarations under 37 CFR 1.132 set forth below should be sufficient under the patent laws, rules and procedures as related in the MPEP to overcome the rejection?
10/15/2003 USPTO Reg. Exam.
19 Afternoon Session (Nbr. 456 Ser. 203)
(A)
(B) (C)
(D)
(E)
An uncontradicted declaration by A asserting that the subject matter relied on by the examiner in reference P constitutes A’s sole invention, with the result that Reference P is not available as prior art against the claim.
A declaration by A asserting that “the claimed amplifier has satisfied a long-felt need in the art.”
A declaration by A and accompanying copies of competitors’ advertisements which conclusively show that those competitors have exactly copied appellant’s commercial embodiment of the claimed amplifier.
A declaration by A and supporting documentation establishing that ever since the filing date of A’s application, sales of the commercial embodiment of A’s claimed amplifier have consistently constituted ninety percent or more of the relevant signal amplifier market in the United States.
None of the above.
35. ANSWER: (A) is the correct answer. See MPEP § 716.10, which states "Under certain circumstances an affidavit or declaration may be submitted which attempts to attribute an activity, a reference or part of a reference to the applicant. If successful, the activity or the reference is no longer applicable. . . . An uncontradicted 'unequivocal statement' from the applicant regarding the subject matter disclosed in an article, patent, or published application will be accepted as establishing inventorship. In re DeBaun, 687 F.2d 459, 463, 214 USPQ 933, 936 (CCPA 1982)." (B) is insufficient in the absence of "objective evidence that an art recognized problem existed in the art for a long period of time without solution." MPEP § 716.04 under heading "THE CLAIMED INVENTION MUST SATISFY A LONG-FELT NEED WHICH WAS RECOGNIZED, PERSISTENT, AND NOT SOLVED BY OTHERS." Regarding (C), see MPEP § 716.06: "[M]ore than the mere fact of copying is necessary to make that action significant because copying may be attributable to other factors such as a lack of concern for patent property or contempt for the patentees ability to enforce the patent. Cable Electric Products, Inc. v. Genmark, Inc., 770 F.2d 1015, 226 USPQ 881 (Fed. Cir. 1985)."
36. An application naming X and Y as joint inventors, filed on April 3, 2002, has a single pending claim, and does not claim the benefit of any earlier application. Which, if any, of the following items of prior art that have been relied on in various rejections of the claim may be overcome by a suitable affidavit under 37 CFR 1.131 in accordance with the patent laws, rules and procedures as related in the MPEP?
(A) A U.S. patent to G that issued on March 27, 2001, has an effective U.S. filing date of January 4, 2000, and does not claim the “same patentable invention” (as defined in 37 CFR 1.601(n)) as the rejected claim.
(B) A U.S. patent to P that issued on June 5, 2001, has an effective U.S. filing date of February 1, 2000, and includes a claim that is identical to the rejected claim.
(C) A journal article to H published on December 10, 2001, and characterized in the application as “describ[ing] the prior art.”
(D) A foreign patent issued to X and Y on November 7, 2001, which claims the same subject matter as the rejected claim and is based on an application filed on January 3, 2001.
(E) None of the above.
36. ANSWER: The correct answer is (E), "None of the above." (A) is incorrect because the G patent is a 35 U.S.C. § 102(b) statutory bar. See MPEP § 715 under heading "Situations Where 37 CFR 1.131 Affidavits Or Declarations Are Inappropriate," states "(A) Where the reference publication date is more than 1 year prior to applicant’s or patent owner’s effective filing date. Such a reference is a 'statutory bar' under 35 U.S.C. 102(b) as referenced in 37 CFR 1.131(a)(2)." (B) is incorrect because 37 CFR § 1.131 expressly provides that prior invention may not be established under the rule "if the rejection is based upon a U.S. patent or U.S. patent application publication of a pending or patented application to another or others which claims the same patentable invention as defined in 37 CFR 1.601(n)." 37 CFR § 1.131(a)(1). Regarding (C), see MPEP §715, under heading " Situ
37. The specification of an application does not disclose the utility of the claimed composition. In fact, the claimed invention is useful for shrinking a specific class of tumors. In a first Office action, the primary examiner has properly determined that the claims lack utility, and has rejected all of the composition claims under the first paragraph of 35 USC 112 as lacking utility. Which of the following responses is in accord with the USPTO rules and the procedures of the MPEP for persuading the examiner that the rejection is improper?
(A) (B) (C) (D)
(E)
Explain that the rejection is statutorily improper because the first paragraph of section 112 is concerned with enablement and written description issues and therefore does not support a rejection for lack of utility.
Point out that the rejection is based on an erroneous finding by the examiner because the specification, in fact, clearly discloses that the composition in question possesses “useful biological” properties.
Show that the rejection is improper by filing probative evidence that the claimed composition has unambiguously proven to be useful for shrinking a specific class of tumors.
File declarations by persons with ordinary skill in the art stating that they would immediately appreciate that the claimed composition is useful for shrinking a specific class of tumors due to the fact that similar compositions having the same characteristics as applicant’s claimed composition were known to be effective for this purpose.
Argue that the rejection is improper because the examiner has failed to present evidence in support of his position that the claimed composition has no utility.
37. ANSWER: (D) is most correct answer. As explained at MPEP § 2107.02, II, B, under the heading “No Statement of Utility for the Claimed Invention in the Specification Does Not Per Se Negate Utility,” the fact that a specification does not contain a statement of utility for the claimed invention does not per se negate utility. This is because a claimed invention may have a well-established utility, and an invention has a well-established utility if (i) a person of ordinary skill in the art would immediately appreciate why the invention is useful based on the characteristics of the invention and (ii) the utility is specific, substantial, and credible. In this case, the declarations specify a specific substantial and credible utility and explain why the declarants (i.e., persons of ordinary skill in the art) would immediately appreciate that the applicant’s claimed composition would possess this utility.
38. A registered practitioner properly recorded an assignment document for application A identifying XYZ Company as the assignee. The document assigns to XYZ Company the “subject matter claimed in Application A.” A proper restriction requirement was made by a primary examiner in application A between two distinct inventions, and the practitioner elected to prosecute one of the inventions. Application A was prosecuted, and later became abandoned. Before the abandonment date of application A, the practitioner filed a complete application B as a proper divisional application of application A. Application B claimed the nonelected invention of Application A, and was published as a U.S. application publication. XYZ Company remains the assignee of application A. What must the practitioner do in accordance with the patent laws, rules and procedures as related in the MPEP to ensure that XYZ Company is listed as the assignee on the face of any patent issuing from application B?
(A) File a proper assignment document in application B identifying XYZ Company as the assignee.
(B) File a proper assignment document in application B identifying XYZ Company as the assignee, and confirm that USPTO’s bibliographic data for application B identifies XYZ Company as the assignee by checking the filing receipt for application B, the U.S. application publication of application B, or the USPTO’s Patent Application Information Retrieval (PAIR) system data for application B, depending on when the practitioner filed the assignment document in application B.
(C) Confirm that XYZ Company is identified as the assignee on the U.S. application publication of application B.
(D) File a proper assignment document in application B identifying XYZ Company as the assignee, and confirm that XYZ Company is identified as the assignee on the U.S. application publication of application B.
(E) Upon allowance of application B, the practitioner must identify XYZ Company as the assignee in the appropriate space on the Issue Fee Transmittal form for specifying the assignee for application B.
38. ANSWER: (E) is the most correct answer. MPEP §§ 306 and 307. MPEP § 306 states, “In the case of a division or continuation application, a prior assignment recorded against the original application is applied to the division or continuation application because the assignment recorded against the original application gives the assignee rights to the subject matter common to both applications.” MPEP § 307 states, “Irrespective of whether the assignee participates in the prosecution of the application, the patent issues to the assignee if so indicated on the Issue Fee Transmittal form PTOL-85B. Unless an assignee's name and address are identified in item 3 of the Issue Fee Transmittal form PTOL-85B, the patent will issue to the applicant. Assignment data printed on the patent will be based solely on the information so supplied.” A new assignment document need not be recorded for a divisional or continuation application where the assignment recorded in the parent application remains the same. Accordingly, (A), (B) and (D) are incorrect. In addition, (A), (B) and (D) are incorrect because unless an assignee’s name and address are identified in item 3 of PTOL-85B, the patent will issued to the applicant and the assignee information, even if recorded, will not appear on the patent. (C) is incorrect for the same reason. (B) is also incorrect.
39. AninternationalapplicationisfiledintheUnitedStatesReceivingOfficeon September 18, 2002. In accordance with the PCT and USPTO rules and the procedures set forth in the MPEP, which of the following will result in the application not being accorded an international filing date of September 18, 2002?
(A)The description and claims are in German.
(B)The request is signed by an attorney rather than an applicant
(C) The sole applicant is a Canadian resident and national. (D) The application does not contain a claim.
(E) The application is not accompanied by any fees.
39. ANSWER: The correct answer is (D). PCT Article 11(1)(iii)(e); 35 U.S.C. § 363; 37 CFR § 1.431(a); MPEP § 1810. Under PCT Article 11(1)(iii)(e) to be accorded an international filing date an application must have “a part which on the face of it appears to be a claim or claims.”
40. Applicant files a patent application in Japan on January 5, 2000. Applicant files a PCT international application designating the United States on January 5, 2001, based on the Japanese application. The international application is published in English on July 5, 2001. The international application enters the national stage in the United States on September 5, 2001. The USPTO publishes the application on June 6, 2002. The application issues as a United States patent on December 3, 2002. What is its earliest possible 35 USC 102(e) prior art date for the application published by the United States, in view of the amendment to Title 35 by the American Inventors Protection Act of 1999 and the Intellectual Property and High Technology Technical Amendments Act of 2002?
(A) January 5, 2000.
(B) January 5, 2001.
(C) July 5, 2001.
(D) June 6, 2002.
(E) December 3, 2002.
40. ANSWER: (B) is the most correct answer. 35 U.S.C. § 102(e)(1) provides that a US published application of a national stage of an international application filed on or after November 29, 2000 has a prior art effect as of its international filing date, if the international application designated the United States, and was published in English. Because in the above fact pattern, the international application designated the United States and was published in English, and was filed on or after November 29, 2000, the USPTO published application is entitled to its international filing date of January 5, 2001 for prior art purposes under 35 U.S.C. § 102(e)(1). See Example 4 of MPEP § 706.02(f)(1). (A) is wrong because the Japanese filing date is relevant under 35 U.S.C. § 119(a) only for priority and not prior art purposes. (C) and (E) are wrong because they recite prior art dates that are later than January 5, 2001. (D) is wrong because the prior art date under 35 U.S.C. § 102(e)(1) is earlier than the application publication date, June 6, 2002.
31. In accordance with the patent laws, rules and procedures as related in the MPEP, for a nonprovisional application to receive a filing date in the USPTO under 37 CFR 1.53(b), all of the following must be filed except:
(A) An oath or declaration executed by applicant pursuant to 37 CFR 1.63.
(B) A specification as prescribed by the first paragraph of 35 USC 112.
(C) A description pursuant to 37 CFR 1.71.
(D) At least one claim pursuant to 37 CFR 1.75.
(E) A drawing when required by 37 CFR 1.81(a).
31. ANSWER: (A) is the most correct answer. 35 U.S.C. § 111; 37 CFR § 1.53; MPEP § 601.01 As provided in 37 CFR § 1.53(f) and MPEP § 601.01(a), the oath or declaration for an application filed under 37 CFR 1.53(b) can be submitted after the filing date. (B), (C), (D) and (E) are incorrect. 37 CFR § 53(b); MPEP § 601.01. 37 CFR § 1.53(b) provides that a filing date is granted on the date on which a specification as prescribed by 35 U.S.C. § 112 containing a description pursuant to § 1.71 and at least one claim pursuant to § 1.75, and any drawing required by § 1.81(a) are filed in the Office. Thus, (B), (C), (D) and (E) are needed to obtain a filing date.
32. Determine which of the following documents, if any, must also contain a separate verification statement in accordance with the patent laws, rules and procedures as related in the MPEP.
(A) A request to correct inventorship in a pending application.
(B) A petition to make an application special.
(C) A claim for foreign priority.
(D) A substitute specification.
(E) None of the above.
32. ANSWER: (E) is the most correct answer. MPEP § 410 states that the certification requirement set forth in 37 CFR § 10.18(b) “has permitted the USPTO to eliminate the separate verification requirement previously contained in 37 CFR ...1.48 [correction of inventorship in a patent application], ...1.55 [claim for foreign priority], ...1.102 [petition to make an application special], [and] ... 1.125 [substitute specification].”
33. A registered practitioner files an application on the client’s discovery that adding silica to a known plastic composition containing the flame retardant, X, results in increased flame retardance. The application claims a composition comprising the known plastic composition containing X and also silica. The primary examiner rejects the claim on the basis that applicant admits that X was a known flame retardant and that there is no evidence of improved flame retardance. In accordance with the patent laws, rules and procedures as related in the MPEP, a proper reply could include which of the following argument(s) to rebut and overcome the rejection?
10/15/2003 USPTO Reg. Exam.
18 Afternoon Session (Nbr. 456 Ser. 203)
(A) (B)
(C)
(D) (E)
The examiner cannot rely on admitted prior art.
The examiner has not shown that the prior art appreciated applicant’s discovery of silica to be a flame retardant.
The examiner has not made out a prima facie case of obviousness due to lack of motivation in the prior art or in the knowledge generally available to one of ordinary skill in the art for adding silica to the known plastic composition.
The applicant does not have to show an improved or unexpected result for the claimed invention.
(C) and (D).
33. ANSWER: (E) is the most correct answer. MPEP §§ 2141, 2142, 2143 and 2143.01. Regarding (C) MPEP 2142 under the heading “ESTABLISHING A PRIMA FACIE CASE OF OBVIOUSNESS,” states “To establish a prima facie case of obviousness, three basic criteria must be met. First, there must be some suggestion or motivation, either in the references themselves or in the knowledge generally available to one of ordinary skill in the art, to modify the reference or to combine reference teachings. Second, there must be a reasonable expectation of success. Finally, the prior art reference (or references when combined) must teach or suggest all the claim limitations. The teaching or suggestion to make the claimed combination and the reasonable expectation of success must both be found in the prior art, and not based on applicant’s disclosure. In re Vaeck, 947 F.2d 488, 20 USPQ2d 1438 (Fed. Cir. 1991). See MPEP § 2143 for decisions pertinent to each of these criteria.” MPEP § 2143 states the same criteria, and further “The teaching or suggestion to make the claimed combination and the reasonable expectation of success must both be found in the prior art, not in applicant's disclosure. In re Vaeck, 947 F.2d 488, 20 USPQ2d 1438 (Fed. Cir. 1991).” Regarding motivation, MPEP 2143.01 states “Obviousness can only be established by combining or modifying the teachings of the prior art to produce the claimed invention where there is some teaching, suggestion, or motivation to do so found either explicitly or implicitly in the references themselves or in the knowledge generally available to one of ordinary skill in the art. ‘The test for an implicit showing is what the combined teachings, knowledge of one of ordinary skill in the art, and the nature of the problem to be solved as a whole would have suggested to those of ordinary skill in the art.’
34. In accordance with the patent laws, rules and procedures as related in the MPEP , which of the following paper is precluded from receiving the benefit of a certificate of mailing or transmission under 37 CFR 1.8?
(A) An amendment, replying to an Office action setting a period for reply, transmitted by mail with a certificate of mailing to the USPTO from a foreign country.
(B) An amendment, replying to an Office action setting a period for reply, transmitted
by facsimile with a certificate of transmission to the USPTO from a foreign
country.
(C) An information disclosure statement (IDS) under 37 CFR 1.97 and 1.98
transmitted after the first Office action.
(D) A request for continued examination (RCE) under 37 CFR 1.114.
(E) An appeal brief.
34. ANSWER: (A) is the most correct answer. See MPEP § 512, which states “The Certificate of Mailing procedure does not apply to papers mailed in a foreign country.” (B) is not correct. See MPEP § 512. Certificate of transmission procedure applies to correspondence transmitted to the Office from a foreign country and an amendment is not prohibited from being transmitted by facsimile and is not precluded from receiving the benefits under 37 CFR § 1.8. (C) is not correct. See MPEP § 609, subsection “Time for Filing.” An IDS will be considered to have been filed on the date of mailing if accompanied by a properly executed certificate of mailing or facsimile transmission under 37 CFR § 1.8. (D) is not correct. See MPEP § 706.07(h) Comparison Chart. An RCE is entitled to the benefit of a certificate of mailing or transmission under 37 CFR § 1.8. (E) is not correct. See MPEP § 1206. An appeal brief is entitled to the benefit of a certificate of mailing or transmission under 37 CFR § 1.8 because it is required to be filed in the Office within a set time period which is 2 months from the date of appeal.
35. The sole claim in an application filed by A and having an effective filing date of June 5, 2002, recites an electrical signal amplifier comprising a plurality of germanium transistors connected together in a particular configuration. The claim is rejected under 35 USC 103(a) as being obviousness over a primary nonpatent reference publication (Reference P) in view of a secondary nonpatent reference publication (Reference S). Reference P has an effective date of April 3, 2002, and names A and B as the authors. Reference S has an effective date of December 10, 2001, and names C as the sole author. Reference P discloses an electrical signal amplifier including a plurality of silicon transistors connected together in the same configuration as that set forth in the claim. Reference S discloses a signal amplifier employing germanium transistors connected in a configuration different from the claimed configuration. The applicant does not deny that the references render the claimed subject matter prima facie obvious. Which, if any, of the declarations under 37 CFR 1.132 set forth below should be sufficient under the patent laws, rules and procedures as related in the MPEP to overcome the rejection?
10/15/2003 USPTO Reg. Exam.
19 Afternoon Session (Nbr. 456 Ser. 203)
(A)
(B) (C)
(D)
(E)
An uncontradicted declaration by A asserting that the subject matter relied on by the examiner in reference P constitutes A’s sole invention, with the result that Reference P is not available as prior art against the claim.
A declaration by A asserting that “the claimed amplifier has satisfied a long-felt need in the art.”
A declaration by A and accompanying copies of competitors’ advertisements which conclusively show that those competitors have exactly copied appellant’s commercial embodiment of the claimed amplifier.
A declaration by A and supporting documentation establishing that ever since the filing date of A’s application, sales of the commercial embodiment of A’s claimed amplifier have consistently constituted ninety percent or more of the relevant signal amplifier market in the United States.
None of the above.
35. ANSWER: (A) is the correct answer. See MPEP § 716.10, which states "Under certain circumstances an affidavit or declaration may be submitted which attempts to attribute an activity, a reference or part of a reference to the applicant. If successful, the activity or the reference is no longer applicable. . . . An uncontradicted 'unequivocal statement' from the applicant regarding the subject matter disclosed in an article, patent, or published application will be accepted as establishing inventorship. In re DeBaun, 687 F.2d 459, 463, 214 USPQ 933, 936 (CCPA 1982)." (B) is insufficient in the absence of "objective evidence that an art recognized problem existed in the art for a long period of time without solution." MPEP § 716.04 under heading "THE CLAIMED INVENTION MUST SATISFY A LONG-FELT NEED WHICH WAS RECOGNIZED, PERSISTENT, AND NOT SOLVED BY OTHERS." Regarding (C), see MPEP § 716.06: "[M]ore than the mere fact of copying is necessary to make that action significant because copying may be attributable to other factors such as a lack of concern for patent property or contempt for the patentees ability to enforce the patent. Cable Electric Products, Inc. v. Genmark, Inc., 770 F.2d 1015, 226 USPQ 881 (Fed. Cir. 1985)."
36. An application naming X and Y as joint inventors, filed on April 3, 2002, has a single pending claim, and does not claim the benefit of any earlier application. Which, if any, of the following items of prior art that have been relied on in various rejections of the claim may be overcome by a suitable affidavit under 37 CFR 1.131 in accordance with the patent laws, rules and procedures as related in the MPEP?
(A) A U.S. patent to G that issued on March 27, 2001, has an effective U.S. filing date of January 4, 2000, and does not claim the “same patentable invention” (as defined in 37 CFR 1.601(n)) as the rejected claim.
(B) A U.S. patent to P that issued on June 5, 2001, has an effective U.S. filing date of February 1, 2000, and includes a claim that is identical to the rejected claim.
(C) A journal article to H published on December 10, 2001, and characterized in the application as “describ[ing] the prior art.”
(D) A foreign patent issued to X and Y on November 7, 2001, which claims the same subject matter as the rejected claim and is based on an application filed on January 3, 2001.
(E) None of the above.
36. ANSWER: The correct answer is (E), "None of the above." (A) is incorrect because the G patent is a 35 U.S.C. § 102(b) statutory bar. See MPEP § 715 under heading "Situations Where 37 CFR 1.131 Affidavits Or Declarations Are Inappropriate," states "(A) Where the reference publication date is more than 1 year prior to applicant’s or patent owner’s effective filing date. Such a reference is a 'statutory bar' under 35 U.S.C. 102(b) as referenced in 37 CFR 1.131(a)(2)." (B) is incorrect because 37 CFR § 1.131 expressly provides that prior invention may not be established under the rule "if the rejection is based upon a U.S. patent or U.S. patent application publication of a pending or patented application to another or others which claims the same patentable invention as defined in 37 CFR 1.601(n)." 37 CFR § 1.131(a)(1). Regarding (C), see MPEP §715, under heading " Situ
31. In accordance with the patent laws, rules and procedures as related in the MPEP, for a nonprovisional application to receive a filing date in the USPTO under 37 CFR 1.53(b), all of the following must be filed except:
(A) An oath or declaration executed by applicant pursuant to 37 CFR 1.63.
(B) A specification as prescribed by the first paragraph of 35 USC 112.
(C) A description pursuant to 37 CFR 1.71.
(D) At least one claim pursuant to 37 CFR 1.75.
(E) A drawing when required by 37 CFR 1.81(a).
31. ANSWER: (A) is the most correct answer. 35 U.S.C. § 111; 37 CFR § 1.53; MPEP § 601.01 As provided in 37 CFR § 1.53(f) and MPEP § 601.01(a), the oath or declaration for an application filed under 37 CFR 1.53(b) can be submitted after the filing date. (B), (C), (D) and (E) are incorrect. 37 CFR § 53(b); MPEP § 601.01. 37 CFR § 1.53(b) provides that a filing date is granted on the date on which a specification as prescribed by 35 U.S.C. § 112 containing a description pursuant to § 1.71 and at least one claim pursuant to § 1.75, and any drawing required by § 1.81(a) are filed in the Office. Thus, (B), (C), (D) and (E) are needed to obtain a filing date.
37. The specification of an application does not disclose the utility of the claimed composition. In fact, the claimed invention is useful for shrinking a specific class of tumors. In a first Office action, the primary examiner has properly determined that the claims lack utility, and has rejected all of the composition claims under the first paragraph of 35 USC 112 as lacking utility. Which of the following responses is in accord with the USPTO rules and the procedures of the MPEP for persuading the examiner that the rejection is improper?
(A) (B) (C) (D)
(E)
Explain that the rejection is statutorily improper because the first paragraph of section 112 is concerned with enablement and written description issues and therefore does not support a rejection for lack of utility.
Point out that the rejection is based on an erroneous finding by the examiner because the specification, in fact, clearly discloses that the composition in question possesses “useful biological” properties.
Show that the rejection is improper by filing probative evidence that the claimed composition has unambiguously proven to be useful for shrinking a specific class of tumors.
File declarations by persons with ordinary skill in the art stating that they would immediately appreciate that the claimed composition is useful for shrinking a specific class of tumors due to the fact that similar compositions having the same characteristics as applicant’s claimed composition were known to be effective for this purpose.
Argue that the rejection is improper because the examiner has failed to present evidence in support of his position that the claimed composition has no utility.
37. ANSWER: (D) is most correct answer. As explained at MPEP § 2107.02, II, B, under the heading “No Statement of Utility for the Claimed Invention in the Specification Does Not Per Se Negate Utility,” the fact that a specification does not contain a statement of utility for the claimed invention does not per se negate utility. This is because a claimed invention may have a well-established utility, and an invention has a well-established utility if (i) a person of ordinary skill in the art would immediately appreciate why the invention is useful based on the characteristics of the invention and (ii) the utility is specific, substantial, and credible. In this case, the declarations specify a specific substantial and credible utility and explain why the declarants (i.e., persons of ordinary skill in the art) would immediately appreciate that the applicant’s claimed composition would possess this utility.
32. Determine which of the following documents, if any, must also contain a separate verification statement in accordance with the patent laws, rules and procedures as related in the MPEP.
(A) A request to correct inventorship in a pending application.
(B) A petition to make an application special.
(C) A claim for foreign priority.
(D) A substitute specification.
(E) None of the above.
32. ANSWER: (E) is the most correct answer. MPEP § 410 states that the certification requirement set forth in 37 CFR § 10.18(b) “has permitted the USPTO to eliminate the separate verification requirement previously contained in 37 CFR ...1.48 [correction of inventorship in a patent application], ...1.55 [claim for foreign priority], ...1.102 [petition to make an application special], [and] ... 1.125 [substitute specification].”
38. A registered practitioner properly recorded an assignment document for application A identifying XYZ Company as the assignee. The document assigns to XYZ Company the “subject matter claimed in Application A.” A proper restriction requirement was made by a primary examiner in application A between two distinct inventions, and the practitioner elected to prosecute one of the inventions. Application A was prosecuted, and later became abandoned. Before the abandonment date of application A, the practitioner filed a complete application B as a proper divisional application of application A. Application B claimed the nonelected invention of Application A, and was published as a U.S. application publication. XYZ Company remains the assignee of application A. What must the practitioner do in accordance with the patent laws, rules and procedures as related in the MPEP to ensure that XYZ Company is listed as the assignee on the face of any patent issuing from application B?
(A) File a proper assignment document in application B identifying XYZ Company as the assignee.
(B) File a proper assignment document in application B identifying XYZ Company as the assignee, and confirm that USPTO’s bibliographic data for application B identifies XYZ Company as the assignee by checking the filing receipt for application B, the U.S. application publication of application B, or the USPTO’s Patent Application Information Retrieval (PAIR) system data for application B, depending on when the practitioner filed the assignment document in application B.
(C) Confirm that XYZ Company is identified as the assignee on the U.S. application publication of application B.
(D) File a proper assignment document in application B identifying XYZ Company as the assignee, and confirm that XYZ Company is identified as the assignee on the U.S. application publication of application B.
(E) Upon allowance of application B, the practitioner must identify XYZ Company as the assignee in the appropriate space on the Issue Fee Transmittal form for specifying the assignee for application B.
38. ANSWER: (E) is the most correct answer. MPEP §§ 306 and 307. MPEP § 306 states, “In the case of a division or continuation application, a prior assignment recorded against the original application is applied to the division or continuation application because the assignment recorded against the original application gives the assignee rights to the subject matter common to both applications.” MPEP § 307 states, “Irrespective of whether the assignee participates in the prosecution of the application, the patent issues to the assignee if so indicated on the Issue Fee Transmittal form PTOL-85B. Unless an assignee's name and address are identified in item 3 of the Issue Fee Transmittal form PTOL-85B, the patent will issue to the applicant. Assignment data printed on the patent will be based solely on the information so supplied.” A new assignment document need not be recorded for a divisional or continuation application where the assignment recorded in the parent application remains the same. Accordingly, (A), (B) and (D) are incorrect. In addition, (A), (B) and (D) are incorrect because unless an assignee’s name and address are identified in item 3 of PTOL-85B, the patent will issued to the applicant and the assignee information, even if recorded, will not appear on the patent. (C) is incorrect for the same reason. (B) is also incorrect.
33. A registered practitioner files an application on the client’s discovery that adding silica to a known plastic composition containing the flame retardant, X, results in increased flame retardance. The application claims a composition comprising the known plastic composition containing X and also silica. The primary examiner rejects the claim on the basis that applicant admits that X was a known flame retardant and that there is no evidence of improved flame retardance. In accordance with the patent laws, rules and procedures as related in the MPEP, a proper reply could include which of the following argument(s) to rebut and overcome the rejection?
10/15/2003 USPTO Reg. Exam.
18 Afternoon Session (Nbr. 456 Ser. 203)
(A) (B)
(C)
(D) (E)
The examiner cannot rely on admitted prior art.
The examiner has not shown that the prior art appreciated applicant’s discovery of silica to be a flame retardant.
The examiner has not made out a prima facie case of obviousness due to lack of motivation in the prior art or in the knowledge generally available to one of ordinary skill in the art for adding silica to the known plastic composition.
The applicant does not have to show an improved or unexpected result for the claimed invention.
(C) and (D).
33. ANSWER: (E) is the most correct answer. MPEP §§ 2141, 2142, 2143 and 2143.01. Regarding (C) MPEP 2142 under the heading “ESTABLISHING A PRIMA FACIE CASE OF OBVIOUSNESS,” states “To establish a prima facie case of obviousness, three basic criteria must be met. First, there must be some suggestion or motivation, either in the references themselves or in the knowledge generally available to one of ordinary skill in the art, to modify the reference or to combine reference teachings. Second, there must be a reasonable expectation of success. Finally, the prior art reference (or references when combined) must teach or suggest all the claim limitations. The teaching or suggestion to make the claimed combination and the reasonable expectation of success must both be found in the prior art, and not based on applicant’s disclosure. In re Vaeck, 947 F.2d 488, 20 USPQ2d 1438 (Fed. Cir. 1991). See MPEP § 2143 for decisions pertinent to each of these criteria.” MPEP § 2143 states the same criteria, and further “The teaching or suggestion to make the claimed combination and the reasonable expectation of success must both be found in the prior art, not in applicant's disclosure. In re Vaeck, 947 F.2d 488, 20 USPQ2d 1438 (Fed. Cir. 1991).” Regarding motivation, MPEP 2143.01 states “Obviousness can only be established by combining or modifying the teachings of the prior art to produce the claimed invention where there is some teaching, suggestion, or motivation to do so found either explicitly or implicitly in the references themselves or in the knowledge generally available to one of ordinary skill in the art. ‘The test for an implicit showing is what the combined teachings, knowledge of one of ordinary skill in the art, and the nature of the problem to be solved as a whole would have suggested to those of ordinary skill in the art.’
39. AninternationalapplicationisfiledintheUnitedStatesReceivingOfficeon September 18, 2002. In accordance with the PCT and USPTO rules and the procedures set forth in the MPEP, which of the following will result in the application not being accorded an international filing date of September 18, 2002?
(A)The description and claims are in German.
(B)The request is signed by an attorney rather than an applicant
(C) The sole applicant is a Canadian resident and national. (D) The application does not contain a claim.
(E) The application is not accompanied by any fees.
39. ANSWER: The correct answer is (D). PCT Article 11(1)(iii)(e); 35 U.S.C. § 363; 37 CFR § 1.431(a); MPEP § 1810. Under PCT Article 11(1)(iii)(e) to be accorded an international filing date an application must have “a part which on the face of it appears to be a claim or claims.”
34. In accordance with the patent laws, rules and procedures as related in the MPEP , which of the following paper is precluded from receiving the benefit of a certificate of mailing or transmission under 37 CFR 1.8?
(A) An amendment, replying to an Office action setting a period for reply, transmitted by mail with a certificate of mailing to the USPTO from a foreign country.
(B) An amendment, replying to an Office action setting a period for reply, transmitted
by facsimile with a certificate of transmission to the USPTO from a foreign
country.
(C) An information disclosure statement (IDS) under 37 CFR 1.97 and 1.98
transmitted after the first Office action.
(D) A request for continued examination (RCE) under 37 CFR 1.114.
(E) An appeal brief.
34. ANSWER: (A) is the most correct answer. See MPEP § 512, which states “The Certificate of Mailing procedure does not apply to papers mailed in a foreign country.” (B) is not correct. See MPEP § 512. Certificate of transmission procedure applies to correspondence transmitted to the Office from a foreign country and an amendment is not prohibited from being transmitted by facsimile and is not precluded from receiving the benefits under 37 CFR § 1.8. (C) is not correct. See MPEP § 609, subsection “Time for Filing.” An IDS will be considered to have been filed on the date of mailing if accompanied by a properly executed certificate of mailing or facsimile transmission under 37 CFR § 1.8. (D) is not correct. See MPEP § 706.07(h) Comparison Chart. An RCE is entitled to the benefit of a certificate of mailing or transmission under 37 CFR § 1.8. (E) is not correct. See MPEP § 1206. An appeal brief is entitled to the benefit of a certificate of mailing or transmission under 37 CFR § 1.8 because it is required to be filed in the Office within a set time period which is 2 months from the date of appeal.
40. Applicant files a patent application in Japan on January 5, 2000. Applicant files a PCT international application designating the United States on January 5, 2001, based on the Japanese application. The international application is published in English on July 5, 2001. The international application enters the national stage in the United States on September 5, 2001. The USPTO publishes the application on June 6, 2002. The application issues as a United States patent on December 3, 2002. What is its earliest possible 35 USC 102(e) prior art date for the application published by the United States, in view of the amendment to Title 35 by the American Inventors Protection Act of 1999 and the Intellectual Property and High Technology Technical Amendments Act of 2002?
(A) January 5, 2000.
(B) January 5, 2001.
(C) July 5, 2001.
(D) June 6, 2002.
(E) December 3, 2002.
40. ANSWER: (B) is the most correct answer. 35 U.S.C. § 102(e)(1) provides that a US published application of a national stage of an international application filed on or after November 29, 2000 has a prior art effect as of its international filing date, if the international application designated the United States, and was published in English. Because in the above fact pattern, the international application designated the United States and was published in English, and was filed on or after November 29, 2000, the USPTO published application is entitled to its international filing date of January 5, 2001 for prior art purposes under 35 U.S.C. § 102(e)(1). See Example 4 of MPEP § 706.02(f)(1). (A) is wrong because the Japanese filing date is relevant under 35 U.S.C. § 119(a) only for priority and not prior art purposes. (C) and (E) are wrong because they recite prior art dates that are later than January 5, 2001. (D) is wrong because the prior art date under 35 U.S.C. § 102(e)(1) is earlier than the application publication date, June 6, 2002.
35. The sole claim in an application filed by A and having an effective filing date of June 5, 2002, recites an electrical signal amplifier comprising a plurality of germanium transistors connected together in a particular configuration. The claim is rejected under 35 USC 103(a) as being obviousness over a primary nonpatent reference publication (Reference P) in view of a secondary nonpatent reference publication (Reference S). Reference P has an effective date of April 3, 2002, and names A and B as the authors. Reference S has an effective date of December 10, 2001, and names C as the sole author. Reference P discloses an electrical signal amplifier including a plurality of silicon transistors connected together in the same configuration as that set forth in the claim. Reference S discloses a signal amplifier employing germanium transistors connected in a configuration different from the claimed configuration. The applicant does not deny that the references render the claimed subject matter prima facie obvious. Which, if any, of the declarations under 37 CFR 1.132 set forth below should be sufficient under the patent laws, rules and procedures as related in the MPEP to overcome the rejection?
10/15/2003 USPTO Reg. Exam.
19 Afternoon Session (Nbr. 456 Ser. 203)
(A)
(B) (C)
(D)
(E)
An uncontradicted declaration by A asserting that the subject matter relied on by the examiner in reference P constitutes A’s sole invention, with the result that Reference P is not available as prior art against the claim.
A declaration by A asserting that “the claimed amplifier has satisfied a long-felt need in the art.”
A declaration by A and accompanying copies of competitors’ advertisements which conclusively show that those competitors have exactly copied appellant’s commercial embodiment of the claimed amplifier.
A declaration by A and supporting documentation establishing that ever since the filing date of A’s application, sales of the commercial embodiment of A’s claimed amplifier have consistently constituted ninety percent or more of the relevant signal amplifier market in the United States.
None of the above.
35. ANSWER: (A) is the correct answer. See MPEP § 716.10, which states "Under certain circumstances an affidavit or declaration may be submitted which attempts to attribute an activity, a reference or part of a reference to the applicant. If successful, the activity or the reference is no longer applicable. . . . An uncontradicted 'unequivocal statement' from the applicant regarding the subject matter disclosed in an article, patent, or published application will be accepted as establishing inventorship. In re DeBaun, 687 F.2d 459, 463, 214 USPQ 933, 936 (CCPA 1982)." (B) is insufficient in the absence of "objective evidence that an art recognized problem existed in the art for a long period of time without solution." MPEP § 716.04 under heading "THE CLAIMED INVENTION MUST SATISFY A LONG-FELT NEED WHICH WAS RECOGNIZED, PERSISTENT, AND NOT SOLVED BY OTHERS." Regarding (C), see MPEP § 716.06: "[M]ore than the mere fact of copying is necessary to make that action significant because copying may be attributable to other factors such as a lack of concern for patent property or contempt for the patentees ability to enforce the patent. Cable Electric Products, Inc. v. Genmark, Inc., 770 F.2d 1015, 226 USPQ 881 (Fed. Cir. 1985)."
31. In accordance with the patent laws, rules and procedures as related in the MPEP, for a nonprovisional application to receive a filing date in the USPTO under 37 CFR 1.53(b), all of the following must be filed except:
(A) An oath or declaration executed by applicant pursuant to 37 CFR 1.63.
(B) A specification as prescribed by the first paragraph of 35 USC 112.
(C) A description pursuant to 37 CFR 1.71.
(D) At least one claim pursuant to 37 CFR 1.75.
(E) A drawing when required by 37 CFR 1.81(a).
31. ANSWER: (A) is the most correct answer. 35 U.S.C. § 111; 37 CFR § 1.53; MPEP § 601.01 As provided in 37 CFR § 1.53(f) and MPEP § 601.01(a), the oath or declaration for an application filed under 37 CFR 1.53(b) can be submitted after the filing date. (B), (C), (D) and (E) are incorrect. 37 CFR § 53(b); MPEP § 601.01. 37 CFR § 1.53(b) provides that a filing date is granted on the date on which a specification as prescribed by 35 U.S.C. § 112 containing a description pursuant to § 1.71 and at least one claim pursuant to § 1.75, and any drawing required by § 1.81(a) are filed in the Office. Thus, (B), (C), (D) and (E) are needed to obtain a filing date.
32. Determine which of the following documents, if any, must also contain a separate verification statement in accordance with the patent laws, rules and procedures as related in the MPEP.
(A) A request to correct inventorship in a pending application.
(B) A petition to make an application special.
(C) A claim for foreign priority.
(D) A substitute specification.
(E) None of the above.
32. ANSWER: (E) is the most correct answer. MPEP § 410 states that the certification requirement set forth in 37 CFR § 10.18(b) “has permitted the USPTO to eliminate the separate verification requirement previously contained in 37 CFR ...1.48 [correction of inventorship in a patent application], ...1.55 [claim for foreign priority], ...1.102 [petition to make an application special], [and] ... 1.125 [substitute specification].”
36. An application naming X and Y as joint inventors, filed on April 3, 2002, has a single pending claim, and does not claim the benefit of any earlier application. Which, if any, of the following items of prior art that have been relied on in various rejections of the claim may be overcome by a suitable affidavit under 37 CFR 1.131 in accordance with the patent laws, rules and procedures as related in the MPEP?
(A) A U.S. patent to G that issued on March 27, 2001, has an effective U.S. filing date of January 4, 2000, and does not claim the “same patentable invention” (as defined in 37 CFR 1.601(n)) as the rejected claim.
(B) A U.S. patent to P that issued on June 5, 2001, has an effective U.S. filing date of February 1, 2000, and includes a claim that is identical to the rejected claim.
(C) A journal article to H published on December 10, 2001, and characterized in the application as “describ[ing] the prior art.”
(D) A foreign patent issued to X and Y on November 7, 2001, which claims the same subject matter as the rejected claim and is based on an application filed on January 3, 2001.
(E) None of the above.
36. ANSWER: The correct answer is (E), "None of the above." (A) is incorrect because the G patent is a 35 U.S.C. § 102(b) statutory bar. See MPEP § 715 under heading "Situations Where 37 CFR 1.131 Affidavits Or Declarations Are Inappropriate," states "(A) Where the reference publication date is more than 1 year prior to applicant’s or patent owner’s effective filing date. Such a reference is a 'statutory bar' under 35 U.S.C. 102(b) as referenced in 37 CFR 1.131(a)(2)." (B) is incorrect because 37 CFR § 1.131 expressly provides that prior invention may not be established under the rule "if the rejection is based upon a U.S. patent or U.S. patent application publication of a pending or patented application to another or others which claims the same patentable invention as defined in 37 CFR 1.601(n)." 37 CFR § 1.131(a)(1). Regarding (C), see MPEP §715, under heading " Situ
37. The specification of an application does not disclose the utility of the claimed composition. In fact, the claimed invention is useful for shrinking a specific class of tumors. In a first Office action, the primary examiner has properly determined that the claims lack utility, and has rejected all of the composition claims under the first paragraph of 35 USC 112 as lacking utility. Which of the following responses is in accord with the USPTO rules and the procedures of the MPEP for persuading the examiner that the rejection is improper?
(A) (B) (C) (D)
(E)
Explain that the rejection is statutorily improper because the first paragraph of section 112 is concerned with enablement and written description issues and therefore does not support a rejection for lack of utility.
Point out that the rejection is based on an erroneous finding by the examiner because the specification, in fact, clearly discloses that the composition in question possesses “useful biological” properties.
Show that the rejection is improper by filing probative evidence that the claimed composition has unambiguously proven to be useful for shrinking a specific class of tumors.
File declarations by persons with ordinary skill in the art stating that they would immediately appreciate that the claimed composition is useful for shrinking a specific class of tumors due to the fact that similar compositions having the same characteristics as applicant’s claimed composition were known to be effective for this purpose.
Argue that the rejection is improper because the examiner has failed to present evidence in support of his position that the claimed composition has no utility.
37. ANSWER: (D) is most correct answer. As explained at MPEP § 2107.02, II, B, under the heading “No Statement of Utility for the Claimed Invention in the Specification Does Not Per Se Negate Utility,” the fact that a specification does not contain a statement of utility for the claimed invention does not per se negate utility. This is because a claimed invention may have a well-established utility, and an invention has a well-established utility if (i) a person of ordinary skill in the art would immediately appreciate why the invention is useful based on the characteristics of the invention and (ii) the utility is specific, substantial, and credible. In this case, the declarations specify a specific substantial and credible utility and explain why the declarants (i.e., persons of ordinary skill in the art) would immediately appreciate that the applicant’s claimed composition would possess this utility.
33. A registered practitioner files an application on the client’s discovery that adding silica to a known plastic composition containing the flame retardant, X, results in increased flame retardance. The application claims a composition comprising the known plastic composition containing X and also silica. The primary examiner rejects the claim on the basis that applicant admits that X was a known flame retardant and that there is no evidence of improved flame retardance. In accordance with the patent laws, rules and procedures as related in the MPEP, a proper reply could include which of the following argument(s) to rebut and overcome the rejection?
10/15/2003 USPTO Reg. Exam.
18 Afternoon Session (Nbr. 456 Ser. 203)
(A) (B)
(C)
(D) (E)
The examiner cannot rely on admitted prior art.
The examiner has not shown that the prior art appreciated applicant’s discovery of silica to be a flame retardant.
The examiner has not made out a prima facie case of obviousness due to lack of motivation in the prior art or in the knowledge generally available to one of ordinary skill in the art for adding silica to the known plastic composition.
The applicant does not have to show an improved or unexpected result for the claimed invention.
(C) and (D).
33. ANSWER: (E) is the most correct answer. MPEP §§ 2141, 2142, 2143 and 2143.01. Regarding (C) MPEP 2142 under the heading “ESTABLISHING A PRIMA FACIE CASE OF OBVIOUSNESS,” states “To establish a prima facie case of obviousness, three basic criteria must be met. First, there must be some suggestion or motivation, either in the references themselves or in the knowledge generally available to one of ordinary skill in the art, to modify the reference or to combine reference teachings. Second, there must be a reasonable expectation of success. Finally, the prior art reference (or references when combined) must teach or suggest all the claim limitations. The teaching or suggestion to make the claimed combination and the reasonable expectation of success must both be found in the prior art, and not based on applicant’s disclosure. In re Vaeck, 947 F.2d 488, 20 USPQ2d 1438 (Fed. Cir. 1991). See MPEP § 2143 for decisions pertinent to each of these criteria.” MPEP § 2143 states the same criteria, and further “The teaching or suggestion to make the claimed combination and the reasonable expectation of success must both be found in the prior art, not in applicant's disclosure. In re Vaeck, 947 F.2d 488, 20 USPQ2d 1438 (Fed. Cir. 1991).” Regarding motivation, MPEP 2143.01 states “Obviousness can only be established by combining or modifying the teachings of the prior art to produce the claimed invention where there is some teaching, suggestion, or motivation to do so found either explicitly or implicitly in the references themselves or in the knowledge generally available to one of ordinary skill in the art. ‘The test for an implicit showing is what the combined teachings, knowledge of one of ordinary skill in the art, and the nature of the problem to be solved as a whole would have suggested to those of ordinary skill in the art.’
38. A registered practitioner properly recorded an assignment document for application A identifying XYZ Company as the assignee. The document assigns to XYZ Company the “subject matter claimed in Application A.” A proper restriction requirement was made by a primary examiner in application A between two distinct inventions, and the practitioner elected to prosecute one of the inventions. Application A was prosecuted, and later became abandoned. Before the abandonment date of application A, the practitioner filed a complete application B as a proper divisional application of application A. Application B claimed the nonelected invention of Application A, and was published as a U.S. application publication. XYZ Company remains the assignee of application A. What must the practitioner do in accordance with the patent laws, rules and procedures as related in the MPEP to ensure that XYZ Company is listed as the assignee on the face of any patent issuing from application B?
(A) File a proper assignment document in application B identifying XYZ Company as the assignee.
(B) File a proper assignment document in application B identifying XYZ Company as the assignee, and confirm that USPTO’s bibliographic data for application B identifies XYZ Company as the assignee by checking the filing receipt for application B, the U.S. application publication of application B, or the USPTO’s Patent Application Information Retrieval (PAIR) system data for application B, depending on when the practitioner filed the assignment document in application B.
(C) Confirm that XYZ Company is identified as the assignee on the U.S. application publication of application B.
(D) File a proper assignment document in application B identifying XYZ Company as the assignee, and confirm that XYZ Company is identified as the assignee on the U.S. application publication of application B.
(E) Upon allowance of application B, the practitioner must identify XYZ Company as the assignee in the appropriate space on the Issue Fee Transmittal form for specifying the assignee for application B.
38. ANSWER: (E) is the most correct answer. MPEP §§ 306 and 307. MPEP § 306 states, “In the case of a division or continuation application, a prior assignment recorded against the original application is applied to the division or continuation application because the assignment recorded against the original application gives the assignee rights to the subject matter common to both applications.” MPEP § 307 states, “Irrespective of whether the assignee participates in the prosecution of the application, the patent issues to the assignee if so indicated on the Issue Fee Transmittal form PTOL-85B. Unless an assignee's name and address are identified in item 3 of the Issue Fee Transmittal form PTOL-85B, the patent will issue to the applicant. Assignment data printed on the patent will be based solely on the information so supplied.” A new assignment document need not be recorded for a divisional or continuation application where the assignment recorded in the parent application remains the same. Accordingly, (A), (B) and (D) are incorrect. In addition, (A), (B) and (D) are incorrect because unless an assignee’s name and address are identified in item 3 of PTOL-85B, the patent will issued to the applicant and the assignee information, even if recorded, will not appear on the patent. (C) is incorrect for the same reason. (B) is also incorrect.
34. In accordance with the patent laws, rules and procedures as related in the MPEP , which of the following paper is precluded from receiving the benefit of a certificate of mailing or transmission under 37 CFR 1.8?
(A) An amendment, replying to an Office action setting a period for reply, transmitted by mail with a certificate of mailing to the USPTO from a foreign country.
(B) An amendment, replying to an Office action setting a period for reply, transmitted
by facsimile with a certificate of transmission to the USPTO from a foreign
country.
(C) An information disclosure statement (IDS) under 37 CFR 1.97 and 1.98
transmitted after the first Office action.
(D) A request for continued examination (RCE) under 37 CFR 1.114.
(E) An appeal brief.
34. ANSWER: (A) is the most correct answer. See MPEP § 512, which states “The Certificate of Mailing procedure does not apply to papers mailed in a foreign country.” (B) is not correct. See MPEP § 512. Certificate of transmission procedure applies to correspondence transmitted to the Office from a foreign country and an amendment is not prohibited from being transmitted by facsimile and is not precluded from receiving the benefits under 37 CFR § 1.8. (C) is not correct. See MPEP § 609, subsection “Time for Filing.” An IDS will be considered to have been filed on the date of mailing if accompanied by a properly executed certificate of mailing or facsimile transmission under 37 CFR § 1.8. (D) is not correct. See MPEP § 706.07(h) Comparison Chart. An RCE is entitled to the benefit of a certificate of mailing or transmission under 37 CFR § 1.8. (E) is not correct. See MPEP § 1206. An appeal brief is entitled to the benefit of a certificate of mailing or transmission under 37 CFR § 1.8 because it is required to be filed in the Office within a set time period which is 2 months from the date of appeal.
39. AninternationalapplicationisfiledintheUnitedStatesReceivingOfficeon September 18, 2002. In accordance with the PCT and USPTO rules and the procedures set forth in the MPEP, which of the following will result in the application not being accorded an international filing date of September 18, 2002?
(A)The description and claims are in German.
(B)The request is signed by an attorney rather than an applicant
(C) The sole applicant is a Canadian resident and national. (D) The application does not contain a claim.
(E) The application is not accompanied by any fees.
39. ANSWER: The correct answer is (D). PCT Article 11(1)(iii)(e); 35 U.S.C. § 363; 37 CFR § 1.431(a); MPEP § 1810. Under PCT Article 11(1)(iii)(e) to be accorded an international filing date an application must have “a part which on the face of it appears to be a claim or claims.”
35. The sole claim in an application filed by A and having an effective filing date of June 5, 2002, recites an electrical signal amplifier comprising a plurality of germanium transistors connected together in a particular configuration. The claim is rejected under 35 USC 103(a) as being obviousness over a primary nonpatent reference publication (Reference P) in view of a secondary nonpatent reference publication (Reference S). Reference P has an effective date of April 3, 2002, and names A and B as the authors. Reference S has an effective date of December 10, 2001, and names C as the sole author. Reference P discloses an electrical signal amplifier including a plurality of silicon transistors connected together in the same configuration as that set forth in the claim. Reference S discloses a signal amplifier employing germanium transistors connected in a configuration different from the claimed configuration. The applicant does not deny that the references render the claimed subject matter prima facie obvious. Which, if any, of the declarations under 37 CFR 1.132 set forth below should be sufficient under the patent laws, rules and procedures as related in the MPEP to overcome the rejection?
10/15/2003 USPTO Reg. Exam.
19 Afternoon Session (Nbr. 456 Ser. 203)
(A)
(B) (C)
(D)
(E)
An uncontradicted declaration by A asserting that the subject matter relied on by the examiner in reference P constitutes A’s sole invention, with the result that Reference P is not available as prior art against the claim.
A declaration by A asserting that “the claimed amplifier has satisfied a long-felt need in the art.”
A declaration by A and accompanying copies of competitors’ advertisements which conclusively show that those competitors have exactly copied appellant’s commercial embodiment of the claimed amplifier.
A declaration by A and supporting documentation establishing that ever since the filing date of A’s application, sales of the commercial embodiment of A’s claimed amplifier have consistently constituted ninety percent or more of the relevant signal amplifier market in the United States.
None of the above.
35. ANSWER: (A) is the correct answer. See MPEP § 716.10, which states "Under certain circumstances an affidavit or declaration may be submitted which attempts to attribute an activity, a reference or part of a reference to the applicant. If successful, the activity or the reference is no longer applicable. . . . An uncontradicted 'unequivocal statement' from the applicant regarding the subject matter disclosed in an article, patent, or published application will be accepted as establishing inventorship. In re DeBaun, 687 F.2d 459, 463, 214 USPQ 933, 936 (CCPA 1982)." (B) is insufficient in the absence of "objective evidence that an art recognized problem existed in the art for a long period of time without solution." MPEP § 716.04 under heading "THE CLAIMED INVENTION MUST SATISFY A LONG-FELT NEED WHICH WAS RECOGNIZED, PERSISTENT, AND NOT SOLVED BY OTHERS." Regarding (C), see MPEP § 716.06: "[M]ore than the mere fact of copying is necessary to make that action significant because copying may be attributable to other factors such as a lack of concern for patent property or contempt for the patentees ability to enforce the patent. Cable Electric Products, Inc. v. Genmark, Inc., 770 F.2d 1015, 226 USPQ 881 (Fed. Cir. 1985)."
40. Applicant files a patent application in Japan on January 5, 2000. Applicant files a PCT international application designating the United States on January 5, 2001, based on the Japanese application. The international application is published in English on July 5, 2001. The international application enters the national stage in the United States on September 5, 2001. The USPTO publishes the application on June 6, 2002. The application issues as a United States patent on December 3, 2002. What is its earliest possible 35 USC 102(e) prior art date for the application published by the United States, in view of the amendment to Title 35 by the American Inventors Protection Act of 1999 and the Intellectual Property and High Technology Technical Amendments Act of 2002?
(A) January 5, 2000.
(B) January 5, 2001.
(C) July 5, 2001.
(D) June 6, 2002.
(E) December 3, 2002.
40. ANSWER: (B) is the most correct answer. 35 U.S.C. § 102(e)(1) provides that a US published application of a national stage of an international application filed on or after November 29, 2000 has a prior art effect as of its international filing date, if the international application designated the United States, and was published in English. Because in the above fact pattern, the international application designated the United States and was published in English, and was filed on or after November 29, 2000, the USPTO published application is entitled to its international filing date of January 5, 2001 for prior art purposes under 35 U.S.C. § 102(e)(1). See Example 4 of MPEP § 706.02(f)(1). (A) is wrong because the Japanese filing date is relevant under 35 U.S.C. § 119(a) only for priority and not prior art purposes. (C) and (E) are wrong because they recite prior art dates that are later than January 5, 2001. (D) is wrong because the prior art date under 35 U.S.C. § 102(e)(1) is earlier than the application publication date, June 6, 2002.
36. An application naming X and Y as joint inventors, filed on April 3, 2002, has a single pending claim, and does not claim the benefit of any earlier application. Which, if any, of the following items of prior art that have been relied on in various rejections of the claim may be overcome by a suitable affidavit under 37 CFR 1.131 in accordance with the patent laws, rules and procedures as related in the MPEP?
(A) A U.S. patent to G that issued on March 27, 2001, has an effective U.S. filing date of January 4, 2000, and does not claim the “same patentable invention” (as defined in 37 CFR 1.601(n)) as the rejected claim.
(B) A U.S. patent to P that issued on June 5, 2001, has an effective U.S. filing date of February 1, 2000, and includes a claim that is identical to the rejected claim.
(C) A journal article to H published on December 10, 2001, and characterized in the application as “describ[ing] the prior art.”
(D) A foreign patent issued to X and Y on November 7, 2001, which claims the same subject matter as the rejected claim and is based on an application filed on January 3, 2001.
(E) None of the above.
36. ANSWER: The correct answer is (E), "None of the above." (A) is incorrect because the G patent is a 35 U.S.C. § 102(b) statutory bar. See MPEP § 715 under heading "Situations Where 37 CFR 1.131 Affidavits Or Declarations Are Inappropriate," states "(A) Where the reference publication date is more than 1 year prior to applicant’s or patent owner’s effective filing date. Such a reference is a 'statutory bar' under 35 U.S.C. 102(b) as referenced in 37 CFR 1.131(a)(2)." (B) is incorrect because 37 CFR § 1.131 expressly provides that prior invention may not be established under the rule "if the rejection is based upon a U.S. patent or U.S. patent application publication of a pending or patented application to another or others which claims the same patentable invention as defined in 37 CFR 1.601(n)." 37 CFR § 1.131(a)(1). Regarding (C), see MPEP §715, under heading " Situ
37. The specification of an application does not disclose the utility of the claimed composition. In fact, the claimed invention is useful for shrinking a specific class of tumors. In a first Office action, the primary examiner has properly determined that the claims lack utility, and has rejected all of the composition claims under the first paragraph of 35 USC 112 as lacking utility. Which of the following responses is in accord with the USPTO rules and the procedures of the MPEP for persuading the examiner that the rejection is improper?
(A) (B) (C) (D)
(E)
Explain that the rejection is statutorily improper because the first paragraph of section 112 is concerned with enablement and written description issues and therefore does not support a rejection for lack of utility.
Point out that the rejection is based on an erroneous finding by the examiner because the specification, in fact, clearly discloses that the composition in question possesses “useful biological” properties.
Show that the rejection is improper by filing probative evidence that the claimed composition has unambiguously proven to be useful for shrinking a specific class of tumors.
File declarations by persons with ordinary skill in the art stating that they would immediately appreciate that the claimed composition is useful for shrinking a specific class of tumors due to the fact that similar compositions having the same characteristics as applicant’s claimed composition were known to be effective for this purpose.
Argue that the rejection is improper because the examiner has failed to present evidence in support of his position that the claimed composition has no utility.
37. ANSWER: (D) is most correct answer. As explained at MPEP § 2107.02, II, B, under the heading “No Statement of Utility for the Claimed Invention in the Specification Does Not Per Se Negate Utility,” the fact that a specification does not contain a statement of utility for the claimed invention does not per se negate utility. This is because a claimed invention may have a well-established utility, and an invention has a well-established utility if (i) a person of ordinary skill in the art would immediately appreciate why the invention is useful based on the characteristics of the invention and (ii) the utility is specific, substantial, and credible. In this case, the declarations specify a specific substantial and credible utility and explain why the declarants (i.e., persons of ordinary skill in the art) would immediately appreciate that the applicant’s claimed composition would possess this utility.
38. A registered practitioner properly recorded an assignment document for application A identifying XYZ Company as the assignee. The document assigns to XYZ Company the “subject matter claimed in Application A.” A proper restriction requirement was made by a primary examiner in application A between two distinct inventions, and the practitioner elected to prosecute one of the inventions. Application A was prosecuted, and later became abandoned. Before the abandonment date of application A, the practitioner filed a complete application B as a proper divisional application of application A. Application B claimed the nonelected invention of Application A, and was published as a U.S. application publication. XYZ Company remains the assignee of application A. What must the practitioner do in accordance with the patent laws, rules and procedures as related in the MPEP to ensure that XYZ Company is listed as the assignee on the face of any patent issuing from application B?
(A) File a proper assignment document in application B identifying XYZ Company as the assignee.
(B) File a proper assignment document in application B identifying XYZ Company as the assignee, and confirm that USPTO’s bibliographic data for application B identifies XYZ Company as the assignee by checking the filing receipt for application B, the U.S. application publication of application B, or the USPTO’s Patent Application Information Retrieval (PAIR) system data for application B, depending on when the practitioner filed the assignment document in application B.
(C) Confirm that XYZ Company is identified as the assignee on the U.S. application publication of application B.
(D) File a proper assignment document in application B identifying XYZ Company as the assignee, and confirm that XYZ Company is identified as the assignee on the U.S. application publication of application B.
(E) Upon allowance of application B, the practitioner must identify XYZ Company as the assignee in the appropriate space on the Issue Fee Transmittal form for specifying the assignee for application B.
38. ANSWER: (E) is the most correct answer. MPEP §§ 306 and 307. MPEP § 306 states, “In the case of a division or continuation application, a prior assignment recorded against the original application is applied to the division or continuation application because the assignment recorded against the original application gives the assignee rights to the subject matter common to both applications.” MPEP § 307 states, “Irrespective of whether the assignee participates in the prosecution of the application, the patent issues to the assignee if so indicated on the Issue Fee Transmittal form PTOL-85B. Unless an assignee's name and address are identified in item 3 of the Issue Fee Transmittal form PTOL-85B, the patent will issue to the applicant. Assignment data printed on the patent will be based solely on the information so supplied.” A new assignment document need not be recorded for a divisional or continuation application where the assignment recorded in the parent application remains the same. Accordingly, (A), (B) and (D) are incorrect. In addition, (A), (B) and (D) are incorrect because unless an assignee’s name and address are identified in item 3 of PTOL-85B, the patent will issued to the applicant and the assignee information, even if recorded, will not appear on the patent. (C) is incorrect for the same reason. (B) is also incorrect.
39. AninternationalapplicationisfiledintheUnitedStatesReceivingOfficeon September 18, 2002. In accordance with the PCT and USPTO rules and the procedures set forth in the MPEP, which of the following will result in the application not being accorded an international filing date of September 18, 2002?
(A)The description and claims are in German.
(B)The request is signed by an attorney rather than an applicant
(C) The sole applicant is a Canadian resident and national. (D) The application does not contain a claim.
(E) The application is not accompanied by any fees.
39. ANSWER: The correct answer is (D). PCT Article 11(1)(iii)(e); 35 U.S.C. § 363; 37 CFR § 1.431(a); MPEP § 1810. Under PCT Article 11(1)(iii)(e) to be accorded an international filing date an application must have “a part which on the face of it appears to be a claim or claims.”
40. Applicant files a patent application in Japan on January 5, 2000. Applicant files a PCT international application designating the United States on January 5, 2001, based on the Japanese application. The international application is published in English on July 5, 2001. The international application enters the national stage in the United States on September 5, 2001. The USPTO publishes the application on June 6, 2002. The application issues as a United States patent on December 3, 2002. What is its earliest possible 35 USC 102(e) prior art date for the application published by the United States, in view of the amendment to Title 35 by the American Inventors Protection Act of 1999 and the Intellectual Property and High Technology Technical Amendments Act of 2002?
(A) January 5, 2000.
(B) January 5, 2001.
(C) July 5, 2001.
(D) June 6, 2002.
(E) December 3, 2002.
40. ANSWER: (B) is the most correct answer. 35 U.S.C. § 102(e)(1) provides that a US published application of a national stage of an international application filed on or after November 29, 2000 has a prior art effect as of its international filing date, if the international application designated the United States, and was published in English. Because in the above fact pattern, the international application designated the United States and was published in English, and was filed on or after November 29, 2000, the USPTO published application is entitled to its international filing date of January 5, 2001 for prior art purposes under 35 U.S.C. § 102(e)(1). See Example 4 of MPEP § 706.02(f)(1). (A) is wrong because the Japanese filing date is relevant under 35 U.S.C. § 119(a) only for priority and not prior art purposes. (C) and (E) are wrong because they recite prior art dates that are later than January 5, 2001. (D) is wrong because the prior art date under 35 U.S.C. § 102(e)(1) is earlier than the application publication date, June 6, 2002.
41. A non-final Office action contains, among other things, a restriction requirement between two groups of claims (Group 1 and Group 2). Determine which of the following, if included in a timely reply under 37 CFR 1.111, preserves applicant’s right to petition the Commissioner to review the restriction requirement in accordance with the patent laws, rules and procedures as related in the MPEP.
10/15/2003 USPTO Reg. Exam.
22 Afternoon Session (Nbr. 456 Ser. 203)
(A)
(B) (C)
(D)
(E)
Applicant’s entire reply to the restriction requirement is: “The examiner erred in distinguishing between Group 1 and Group 2, and therefore the restriction requirement is respectfully traversed and no election is being made, in order that applicant’s right to petition the Commissioner to review the restriction requirement is preserved.”
Applicant’s entire reply to the restriction requirement is: “Applicant elects Group1 and respectfully traverses the restriction requirement, because the examiner erred in requiring a restriction between Group 1 and Group 2.” Applicant’s reply distinctly points out detailed reasons why applicant believes the examiner erred in requiring a restriction between Group 1 and Group 2, and additionally sets forth, “Applicant therefore respectfully traverses the restriction requirement and no election is being made, in order that applicant’s right to petition the Commissioner to review the restriction requirement is preserved.” Applicant’s reply distinctly points out detailed reasons why applicant believes the examiner erred in requiring a restriction between Group 1 and Group 2, and additionally sets forth, “Applicant therefore respectfully traverses the restriction requirement and elects Group 2.
None of the above.
41. ANSWER: (D) is the most correct answer. 37 CFR § 1.111(b); MPEP §§ 818.03(a)-(c). MPEP § 818.03(a) states “[a]s shown by the first sentence of 37 CFR 1.143, the traverse to a requirement must be complete as required by 37 CFR 1.111(b) . . . Under this rule, the applicant is required to specifically point out the reasons on which he or she bases his or her conclusions that a requirement to restrict is in error.” An election must be made even if the requirement is traversed. MPEP § 818.03(b). Answer (A) is incorrect since the traversal does not distinctly point out the supposed errors in the examiner’s action, and no election is made. 37 CFR § 1.143. MPEP § 818.03(a) states “[a] mere broad allegation that the requirement is in error does not comply with the requirement of 37 CFR 1.111.” Answer (A) is also incorrect because no election is made. MPEP § 818.03(b) states, “[a]s noted in the second sentence of 37 CFR 1.143, a provisional election must be made even though the requirement is traversed. (B) is incorrect. MPEP § 818.03 since the traversal does not distinctly point out the supposed errors in the examiner’s action. (C) is incorrect since no election is made. See MPEP § 818.03(b) (E) is incorrect because (D) is correct.
42. The primary examiner has rejected claims 1-10 under 35 USC 103(a) as being unpatentable over the Smith patent in view of the Jones reference. Appellant properly argues that there is no motivation to combine the teachings of Smith and Jones. The examiner repeats the rejection of claims 1-10 as being “unpatentable over Smith in view of Jones.” The examiner additionally cites a patent to Brown that was necessary to provide motivation for combining the teachings of Smith and Jones. The examiner does not list Brown in the statement of the rejection. Appellant timely appeals to the Board of Patent Appeals and Interferences, and files a proper appeal brief. The examiner files an examiner’s answer addressing the rejection of claims 1-10 under 35 USC 103(a) as being unpatentable over Smith in view of Jones, and cites Brown in the argument as providing motivation to combine Smith and Jones. In accordance with the patent laws, rules and procedures as related in the MPEP, what will be the most proper decision of the Board?
(A) The Board will affirm the rejection based on Smith and Jones only.
(B) The Board will affirm the rejection based on Smith, Jones and Brown.
(C) The Board will reverse the rejection based on Smith and Jones only.
(D) The Board will reverse the rejection based on Smith, Jones and Brown.
(E) None of the above.
42. ANSWER: (C) is the most correct answer. 37 CFR § 1.193(a)(2); MPEP § 1208.01. If the claimed invention is rendered obvious by Smith in view of Jones and Brown, the statement of rejection must include all three references. Reliance on Brown to support the rejection is a different rejection from a rejection relying only on Smith in view of Jones. In accordance with MPEP § 1208.01, the Board will not consider the teachings of Brown because Brown was used to support the rejection, but was not listed in the statement of the rejection. As stated in MPEP § 1208.01, “Even if the prior art reference is cited to support the rejection in a minor capacity, it should be positively included in the statement of rejection. In re Hoch, 428 F.2d 1341, 1342 n.3, 166 USPQ 406, 407 n.3 (CCPA 1970).” Therefore, (B) and (D) are clearly wrong. (A) is incorrect. The decision cannot affirm the rejection since there is no motivation for combining the teachings of Smith and Jones absent the teachings of Brown. Therefore, the rejection must be reversed, not affirmed.
43. Which of the following statement(s) is in accordance with patent laws, rules and procedures as related in the MPEP regarding filing of an Application Data Sheet (ADS) in the USPTO?
10/15/2003 USPTO Reg. Exam.
23 Afternoon Session (Nbr. 456 Ser. 203)
(A) (B)
(C)
(D) (E)
All non-provisional applications must include an ADS when the application is originally filed.
If an ADS is filed at the same time as an oath or declaration under 37 CFR 1.63 or 1.67 and the information supplied in the two documents is inconsistent, the information provided in the ADS will always govern.
If an ADS is filed at the same time as an oath or declaration under 37 CFR 1.63 or 1.67 and the information supplied in the two documents is inconsistent, the oath or declaration will govern any inconsistency related the claiming of benefit under 35 USC 119(e), 120, 121 or 365(c).
If an ADS is filed after an oath or declaration under 37 CFR 1.63 or 1.67 is filed, and the information supplied in the two documents is inconsistent, the information provided in the ADS will always govern.
The oath or declaration under 37 CFR 1.63 or 1.67 governs inconsistencies with the ADS when the inconsistency concerns setting forth the citizenship of the inventor(s) under 35 USC 115.
43. ANSWER: (E) is the most correct answer. See 37 CFR § 1.76(d)(3); MPEP § 601.05. The oath or declaration filed under 37 CFR §§ 1.63 or 1.67.governs any inconsistency with the Application Data Sheet (ADS) when the inconsistency relates to the setting forth the citizenship of the inventor(s) even if the ADS was filed after the filing of the oath or declaration. (A) is incorrect because an ADS is a voluntary submission in either a provisional application or a nonprovisional application. See 37 CFR 1.76(a). (B) is an incorrect answer because the information related to the naming of inventors 37 CFR § 1.41(a)(1) and the information relating to setting forth citizenship 35 U.S.C. § 115 is governed by the oath or declaration filed under 37 CFR §§ 1.63 or 1.67 regardless if the two documents ar
44. In accordance with the patent laws, rules and procedures as related in the MPEP, which of the following statements regarding claim interpretation is the most correct?
(A) A claim having the transition term “comprising” is limited to only the limitations, elements or steps recited in the claim, and is not inclusive or open-ended of other unrecited elements or steps.
(B) The transition term “consisting essentially of” limits the claim to the limitations recited in the claim and additional elements or steps which do not materially affect the basic and novel characteristics of the claimed invention.
(C) A claim having the transition term “consisting of” is not limited to the elements or steps recited in the claim, but can include elements or steps other than those recited in addition to any impurities ordinarily associated therewith.
(D) A claim which depends from a claim which claims an invention “consisting of” the recited elements or steps can add an element or step to further limit the claimed invention.
(E) All of the above.
44. ANSWER: (B) is the most correct answer. MPEP § 2111.03 (fourth paragraph) states, in reliance upon In re Herz, 537 F.2d 549, 551-52, 190 USPQ 461, 463 (CCPA 1976), that “[t]he transitional phrase ‘consisting essentially of’ limits the scope of a claim to the specified materials or steps ‘and those that do not materially affect the basic and novel characteristic(s)’ of the claimed invention.” (A) is incorrect. The statement is contradicted by MPEP § 2111.03 (second paragraph), which states, in reliance upon Genentech, Inc. v. Chiron Corp., 112, F.3d 495, 501, 42 USPQ2d 1608, 1613 (Fed. Cir. 1997) that “[t]he transition term ‘comprising’, which is synonymous with ‘including,’ ‘containing,’ or ‘characterized by,’ is inclusive or open-ended and does not exclude additional, unrecited elements or method steps. (C) is incorrect. The statement is contradicted by MPEP § 2111.03 (third paragraph), which states, in reliance upon In re Gray, 53 F.2d 520, 11 USPQ 255 (CCPA 1931) that “[t]he transitional phrase ‘consisting of’ excludes any element, step , or ingredient not specified in the claim.” (D) is incorrect. The statement is directly contradicted by MPEP § 2111.03 (third paragraph), which states “[a] claim which depends from a claim which ‘consists of’ the recited elements or steps cannot add an element or step.” (E) is incorrect because (A), (B) and (C) are incorrect.
5. A patent application has claims 1-10 pending. Claims 1 and 7 are independent claims. Claims 2-6 depend directly from claim 1 while claims 8-10 depend directly from claim 7. Claims 1-10 have been twice rejected by the primary examiner under 35 USC 103(a) as being unpatentable over Smith patent in view of Jones patent. The applicant has appealed the rejection to the Board of Patent Appeals and Interferences. In the brief under the “grouping of claims” section, appellant states that each of the claims is separately patentable. In the arguments section of the brief, appellant separately argues only claims 1, 4 and 6. In the examiner’s answer, the examiner disagrees with appellant’s claim grouping because all the claims present a similar issue of patentability. The examiner states that the claims all stand or fall together as a single group. In accordance with the patent laws, rules and procedures as related in the MPEP, which claim(s) must the Board consider separately on the merits?
(A) (B) (C) (D)
(E)
The Board must consider each of claims 1-10 separately on the merits.
The Board must only consider claims 1, 4 and 6 separately on the merits.
The Board must only consider claim 1 separately on the merits.
The Board must consider claim 1 and claim 7 separately on the merits as representative of all the claims on appeal.
The Board must determine which claim is representative of all the claims on appeal and consider only that claim separately on the merits.
45. ANSWER: (B) is the most correct answer. 37 CFR § 1.192(c)(7); MPEP § 1206, under the heading “Appeal Brief Content,” subheading “(7) Grouping of Claims.” 37 CFR § 1.192(c)(7) requires that an appellant perform two affirmative acts in the brief to receive separate consideration of the patentability of a plurality of claims that are subject to the same rejection. The appellant must (1) state that the claims do not stand or fall together and (2) present arguments why the claims subject to the same rejection are separately patentable. Since the appellant here has only performed the two affirmative acts with respect to claims 1, 4 and 6, these are the claims that the Board must consider separately for patentability. The examiner has no input on the grouping of claims. (D) is incorrect inasmuch as § 1.192(c)(7) requires the inclusion of reasons in order to avoid unsupported assertions of separate patentability. See MPEP § 1206, subheading subheading “(7) Grouping of Claims.” Where the grouping of claims section is inconsistent with the arguments section as in the facts of this case, the examiner should have notified the appellant that the brief was in noncompliance as per 37 CFR § 1.192(d). See Ex parte Schier, 21 USPQ2d 1016 (Bd. Pat. App. & Int. 1991); Ex parte Ohsumi, 21 USPQ2d 1020
46. A primary examiner is examining a patent application. The application includes a specification and a single claim to the invention that reads as follows:
1. A building material to be used as an alternative to brick in the construction of a house, said building material comprising compressed refuse, the majority of which is wood.
In the specification, the inventor explains that the wood to be used in the inventive building material should be balsa wood. According to the specification, balsa-containing building material has the advantage of being lighter than brick. In a first Office action mailed to the registered practitioner representing the inventor the single claim was rejected as anticipated under 35 U.S.C. § 102 over Patent A. Patent A issued more than one year before the effective filing date of the application, and teaches a building material to be used as an alternative to brick in the construction of a house comprising compressed refuse, the majority of which is pine. The practitioner replies to the first Office action by arguing that the invention is different from that of Patent A. According to the practitioner, the inventor uses balsa wood, not pine. The claim has not been amended. Which of the following describes how the examiner should proceed in accordance with the patent laws, rules and procedures as related in the MPEP?
(A) The examiner should allow the claim.
(B) The examiner should allow the claim only after including a Reasons for
Allowance pointing out that the inventor argues that her invention is directed to
using balsa wood, not pine.
(C) The examiner should issue a Final Rejection again rejecting the claim as
anticipated under 35 USC102 over Patent A.
(D) The examiner should reopen prosecution and begin anew, this time searching for
a reference that shows a building material containing balsa wood.
(E) The examiner should withdraw the rejection but issue a new Office action this time rejecting the claim under 35 USC 112, second paragraph, because the claim
is broad enough to encompass using pine.
46. ANSWER: (C) is the best answer. 35 U.S.C. § 102; MPEP §§ 2111 and 2131. MPEP § 2131, under the heading, “To Anticipate A Claim, The Reference Must Teach Every Element Of The Claim.” "A claim is anticipated only if each and every element as set forth in the claim is found, either expressly or inherently described, in a single prior art reference." See Verdegaal Bros. v. Union Oil Co. of California, 814 F.2d 628, 631, 2 USPQ2d 1051, 1053 (Fed. Cir. 1987). Here, every element of the claim is found in Patent A. See MPEP 2111, under the heading “Claims Must Be Given Their Broadest Reasonable Interpretation,” where it explained that “[d]uring patent examination, the pending claims must be ‘given the broadest reasonable interpretation consistent with the specification,” and cites In re Prater, 415 F.2d 1393, 1404-05, 162 USPQ 541, 550-51 (CCPA 1969) to explain that "reading a claim in light of the specification, to thereby interpret limitations explicitly recited in the claim, is a quite different thing from 'reading limitations of the specification into a claim,' to thereby narrow the scope of the claim by implicitly adding disclosed limitations which have no express basis in the claim."
47. To rely in a rejection under 35 USC 102(a) on an invention that is known or publicly used in accordance with patent laws, rules and procedures as related in the MPEP, the invention:
10/15/2003 USPTO Reg. Exam.
25 Afternoon Session (Nbr. 456 Ser. 203)
(A) (B)
(C) (D) (E)
must be known or used in NAFTA or WTO member countries.
must be known or used in a NAFTA member country, but only if the filing date of the application is after the effective date of the North American Free Trade Agreement Implementation Act.
must be known or used in this country.
can be known or used in any country.
must be known or used in a WTO member country, but only if the filing date of the application is after the effective date of the implementation of the Uruguay Round (WTO) Agreements Act.
47. ANSWER: (C) is the most correct answer. 35 U.S.C. § 102(a); MPEP § 2132. As set forth in MPEP § 2132, under the heading “II. ‘In This Country,’” subheading “Only Knowledge or Use In The U.S. Can Be Used in a 35 U.S.C. 102(a) Rejection,” states “[t]he knowledge or use relied on in a 35 U.S.C. 102(a) rejection must be knowledge or use ‘in this country.’ Prior knowledge or use which is not present in the United States, even if widespread in a foreign country, cannot be the basis of a rejection under 35 U.S.C. 102(a). In re Ekenstam, 256 F.2d 321, 118 USPQ 349 (CCPA 1958). Note that the changes made to 35 U.S.C.104 by NAFTA (Public Law 103-182) and Uruguay Round Agreements Act (Public Law 103-465) do not modify the meaning of ‘in this country’ as used in 35 U.S.C. 102(a) and thus ‘in this country” still means in the United States for purposes of 35 U.S.C. 102(a) rejections.’” See also MPEP § 706.02(c), “[t]he language ‘in this country’ means in the United States only and does not include other
48. A registered practitioner timely files a petition under 37 CFR 1.181 while the application is pending before the primary examiner to challenge the prematureness of the final rejection that set a shortened statutory period for reply. Assume the petition is filed within two months of the date on the final rejection. What is the next response that should be docketed by the practitioner in accordance with the patent laws, rules and the procedures as related in the MPEP to avoid a penalty or payment of fees?
(A) A reply to the final rejection within 6 months.
(B) A status inquiry 6 months after filing the petition.
(C) A reply to the final rejection within the shortened statutory time period set for
reply in the final rejection.
(D) No reply is necessary until a decision is received on the petition.
(E) All of the above.
48. ANSWER: (C) is correct. 37 CFR § 1.181(f); MPEP §§ 714.13 and 1002: 37 CFR § 1.181(f) provides "The mere filing of a petition will not stay the period for reply to an Examiner's action which may be running against an application, nor act as a stay of other proceedings." MPEP §§ 714.13 states “[i]t should be noted that under 37 CFR 1.181(f), the filing of a 37 CFR 1.181 petition will not stay the period for reply to an examiner's action which may be running against an application.” See also MPEP § 1002. Thus, if a petition to vacate a final rejection as premature is filed within 2 months from the date of the final rejection, the period for reply to the final rejection is not extended even if the petition is not reached for decision within that period. However, if the petition is granted and the applicant has filed an otherwise full reply to the rejection within the period for reply, the case is not abandoned. (C) is correct because the petition does not stay the time for responding to the final and a reply should be filed within the shortened statutory period to avoid fees.
49. A patent specification discloses a personal computer comprising a microprocessor and a random access memory. There is no disclosure in the specification of the minimum amount of storage for the random access memory. In the disclosed preferred embodiment, the microprocessor has a clock speed of 100-200 megahertz. Claims 9 and 10, presented below, are original claims in the application. Claim 11, presented below, was added by amendment after an Office action.
9. A personal computer comprising a microprocessor and a random access memory that includes at least 1 gigabyte of storage.
10. The personal computer of Claim 9, wherein the microprocessor has a clock speed of 100-200 megahertz.
11. The personal computer of Claim 10, wherein the random access memory is greater than 1⁄2 gigabyte of storage.
Which of the following statements is or are in accord with the patent laws, rules and procedures as related in the MPEP regarding the respective claims under the fourth paragraph of 35 USC 112?
(A) (B) (C) (D) (E)
Claim 9 is a proper independent claim, and Claims 10 and 11 are proper dependent claims.
Claim 9 is a proper independent claim, and Claims 10 and 11 are improper dependent claims.
Claim 9 is an improper independent claim, and Claims 10 and 11 are improper dependent claims.
Claim 9 is an improper independent claim, and Claims 10 and 11 are proper dependent claims.
Claim 9 is a proper independent claim, Claim 10 is a proper dependent claim, and Claim 11 is an improper dependent claim.
49. ANSWER: (E) is the most correct answer. 37 CFR § 1.75(c); MPEP § 608.01(n), under the heading “II. Treatment Of Improper Dependent Claims,” 37 CFR § 1.75(c) provides “One or more claims may be presented in dependent form, referring back to and further limiting another claim or claims in the same application.” Claim 9, though broad, is supported by the specification. The minimum memory recited in the claim as original disclosure, is self- supporting. 35 U.S.C. § 112, first paragraph. See MPEP §§ 608.01(l) and 2163. Claim 10 is a proper dependent claim because it depends from and further restricts the scope of a preceding claim. 37 CFR § 1.75(c). Claim 11 is an improper dependent claim because it is inconsistent with and does not further limit the scope of claim 10. Claim 10, depending on Claim 9, has a 1 gigabyte memory minimum, whereas Claim 11 redefines the minimum memory by setting a lower minimum of 1⁄2 gigabyte.
50. In accordance with the patent laws, rules and procedures as related in the MPEP, which of the following facts are required for 35 USC 102(g) to form the basis for an ex parte rejection:
(1) The subject matter at issue has been actually reduced to practice by another before the applicant’s invention.
(2) There has been no abandonment, suppression or concealment.
(3) A U.S. patent application for the subject matter at issue has been filed by another prior to
the filing of the applicant’s application.
(4) A U.S. patent has been granted for the subject matter at issue prior to the filing of the
applicant’s application.
10/15/2003 USPTO Reg. Exam.
27 Afternoon Session (Nbr. 456 Ser. 203)
(A) (B) (C) (D) (E)
Fact (1) only
Fact (2) only
Facts (1) and (2)
Facts (1), (2) and (3) Facts (1), (2), (3) and (4)
50. ANSWER: (C) is the most correct, as a 35 U.S.C. § 102(g) rejection requires actual reduction to practice by another, and lack of abandonment, suppression, or concealment. MPEP § 2138 states “35 U.S.C. 102(g) may form the basis for an ex parte rejection if: (1) the subject matter at issue has been actually reduced to practice by another before the applicant's invention; and (2) there has been no abandonment, suppression or concealment. See, e.g., Amgen, Inc. v.
21
October 15, 2003 Examination Afternoon Session Model Answers